You are on page 1of 96

Climatology

Climatic Zone
1320. At about what geographical latitude as average is assumed for the zone of prevailing westerlies?
A) 10° N.
B) 50° N.
C) 80° N.
D) 30° N.

1321. What is the type, intensity and seasonal variation of precipitation in the equatorial region?
A) Rain showers, hail showers and thunderstorms occur the whole year, but
frequency is highest during two periods: April-May and October-
November.
B) Precipitation is generally in the form of showers but continuous rain
occurs also. The greatest intensity is in July.
C) Warm fronts are common with continuous rain. The frequency is the
same throughout the year
D) Showers of rain or hail occur throughout the year; the frequency is
highest in January.

1324. The reason for the fact, that the Icelandic low is normally deeper in winter than in summer is
that:
A) the strong winds of the north Atlantic in winter are favourable for the
development of lows.
B) the low pressure activity of the sea east of Canada is higher in winter.
C) the temperature contrasts between arctic and equatorial areas are much
greater in winter.
D) converging air currents are of greater intensity in winter.

1328. The lowest relative humidity will be found:


A) at the south pole.
B) between latitudes 30 deg and 40 deg N in July.
C) in equatorial regions.
D) around 30 deg S in January.
Tropical Climatology:
1329. Flying from Dakar to Rio de Janeiro in winter where would you cross the ITCZ?
A) 7 to 120N.
B) 0 to 70N.
C) 7 to 120S.
D) 12 to 180S.

1330. Horse latitudes are:


A) only 40 deg N in the Western Pacific under the sub tropical highs.
B) 40 deg S only.
C) around the heat equator.
D) in the region of the oceanic sub tropical highs in the Northern
Hemisphere.

1331. The NE Monsoon affecting India, blows in ... and gives ... and ... conditions.
A) summer, cool, dry.
B) winter, cool, dry.
C) winter, warm, wet.
D) summer, warm, wet.

1332. Where is the ITCZ during the year?


A) Moves in accordance with the heat equator.
B) Always south of the equator.
C) Doesn't move.
D) Always north of the equator.

1333. An easterly wave is a:


A) wave-like disturbance in the monsoon regime of India, moving from east
to west, with severe convective activity ahead of its trough.
B) small scale wave disturbance in the tropics, moving from east to west,
with severe convective activity ahead of its trough.
C) wave in a trade wind belt, moving from east to west, with severe
convective activity in rear of its trough.
D) disturbance in the higher levels associated with the equatorial easterly
jet, moving from east to west, with severe convective activity in rear of
its trough.

1334. Along the West coast of India the prevailing winds are the:
A) SW monsoon in July and a SE monsoon in January.
B) SE monsoon in July and a SW monsoon in January.
C) NE monsoon in July and a SW monsoon in January.
D) SW monsoon in July and a NE monsoon in January.
1335. Flying from Dakar to Rio de Janeiro, where is the ITCZ in the northern winter?
A) 0 - 7 N
B) 12 - 16 N
C) 8 - 12 N
D) above 8 S

1336. The Doldrums are:


A) another name for the sub-tropical anticyclones.
B) cols between weak fronts encountered in low latitudes.
C) weak inter-tropical convergence zones.
D) associated with light and variable monsoon winds.

1337. The ITCZ is best described as:


A) where cold fronts are formed in the tropics.
B) where the trade winds of the north and southern hemisphere meet.
C) where the west winds meet the sub-tropical high pressure belt.
D) where the Harmattan meets the NE trades in Africa.

1338. What weather conditions are indications of the summer monsoon in India?
A) Thunderstorms, showers of heavy rain.
B) Sandstorms.
C) Stratus clouds and drizzle.
D) Fog.

1339. What is the name of the low level winds between the subtropical high pressure belt and the
ITCZ?
A) Monsoon.
B) Easterly wave.
C) Low level jetstream.
D) Trade winds.

1340. Along the West coast of India the prevailing winds are the:
A) SW monsoon in July and a SE monsoon in January.
B) SE monsoon in July and a SW monsoon in January.
C) NE monsoon in July and a SW monsoon in January.
D) SW monsoon in July and a NE monsoon in January.

1341. In July light easterly upper winds are apparent:


A) all along the Equator.
B) between 20N and 05S over the Pacific Ocean.
C) at 10S.
D) at 20N over Asia and Northern Africa.
1342. The transition from SW to NE monsoon in India occurs in:
A) September, October, November.
B) July, August, September.
C) December, January, February.
D) February, March, April.

1343. What is the Easterly wave?


A) A wave travelling west-east.
B) A wave travelling east-west.
C) A wave travelling south-north.
D) A wave travelling north-south.

1344. The ITCZ reaches its most northerly position:


A) in September over Morocco.
B) in August over Mexico.
C) in July over eastern China.
D) in January over Darwin.

1345. What weather conditions are most likely to affect an approach to Dakar during July?
A) Reduced visibility due to the rising sand of the Harmattan.
B) Dry and clear due to the influence of the Azores high pressure system.
C) Wet and thundery due to the proximity of Intertropical convergence zone
(ITCZ).
D) Generally clear skies - NW trade winds.

1346. Referring to the picture, what wind


would you expect at the position indicated?

A) NE monsoon.
B) Trade wind.
C) Strong westerlies.
D) Roaring forties.

1347. Which one of the following statements regarding the Intertropical convergence zone (ITCZ) is
correct?
A) The ITCZ does not change its position during the course of the year.
B) Thunderstorms seldom occur within the area of the ITCZ.
C) The ITCZ is always associated with a strong jet stream.
D) Frequent and widespread thunderstorms are to be expected within the area of
the ITCZ.
1348. When are the rains most likely in Equatorial Africa?
A) June to July.
B) December to January.
C) March to May, October to November.
D) March to May, August to October.

1349. What name is given to the low level wind system between the subtropical high pressure belt
and the equatorial trough of low pressure (ITCZ)?
A) Doldrums.
B) Monsoon.
C) Westerly winds.
D) Trade winds.

1350.General surface winds in West Africa with ITCZ to the north:


A) East - West.
B) SE trade winds to the north, NE trade winds to the south.
C) NE trade wind to the north, SW monsoon to the south.
D) West - East.

1351. Flying from Bangkok to Bombay, why does the wind at 30.000ft change from 15kts headwind in
winter to a 20kt tailwind in summer?
A) The Equatorial Easterly jet changes direction through 180 degrees.
B) Freak weather conditions experienced on route.
C) The Subtropical jet changes direction through 180 degrees.
D) This is due to local changes in the upper winds due to the movement of the ITCZ.

1352. Which one of the following statements is correct concerning the movement of the ITCZ in the
region of West Africa?
A) It oscillates during the year between the Equator and 10 degrees North.
B) It reaches its maximum southerly position of 5° S in January.
C) It oscillates during the year between 10 degrees North and 10 degrees South.
D) It reaches its maximum northerly position of 15° - 20° N in July.

1353. Horse latitudes are:


A) Dry and clear due to the influence of the Azores high pressure system.
B) Reduced visibility due to the rising sand of the Harmattan.
C) Wet and thundery due to the proximity of Intertropical convergence zone (ITCZ).
D) Generally clear skies - NW trade winds.

1354. The SW Monsoon affecting India, blows in ... and gives ... and ... conditions.
A) summer, cool, dry
B) summer, warm, wet
C) winter, cool, dry
D) winter, warm, wet
1355. Which wind systems converge on the ITCZ, when it lies at the equator?
A) SE trade winds and NE trade winds.
B) SW monsoon and NW trade winds.
C) NW monsoon and SW trade winds.
D) SW monsoon and NW monsoon.

1356Weather conditions at Bombay during January are mainly influenced by the:


A) NW monsoon.
B) SW monsoon.
C) NE monsoon.
D) SE monsoon.

1357. In which of the following bands of latitude is the Intertropical convergence zone most likely to
be encountered in January, between Dakar and Rio de Janeiro?
A) 0° - 7° N.
B) 8° - 12° S.
C) 7° - 12° N.
D) 3° - 8° S.

1358. Which of the following statements concerning the ITCZ is correct?


A) Its furthest displacement from the equator is normally about 45° S.
B) Its associated weather is invariably strong convergence and heavy
cumuliform cloud.
C) In the southern summer it is normally positioned centered south of the
equator.
D) It is normally fed with converging northern and southern trade winds.

1359. Where, during a flight from Marseille to Dakar, in July, may the ITCZ be encountered?
A) Near the Canary Islands.
B) In the vicinity of Dakar.
C) At the latitudes of Gibraltar.
D) At the latitudes of Algeria

1360. Weather conditions at Bombay during early July are mainly influenced by the
A) NE monsoon and the proximity of the ITCZ.
B) SW monsoon.
C) passage of frontal system generated in the south Indian ocean.
D) high incidence of tropical revolving storms originating in the Persian gulf.
1361. During July flights from Bangkok (13° N - 100° E) to Karachi (25° N - 67° E) experience an
average tailwind component of 22 kt. In January the same flights, also operating at FL 370, have an
average headwind of 50 kt.
What is the reason for this difference?
A) The wind components correspond to the seasonal change of the regional
wind system.
B) The flights in January encountered, by chance, very unusual, adverse
conditions.
C) The flights during the summer encountered, by chance, very unusual,
favourable conditions.
D) The flights happen to be in the area of the polar front jet stream.

1363. Which of the following best describes the Intertropical convergence zone?
A) The zone where the trade winds of the northern hemisphere meet those
of the southern hemisphere.
B) The zone where the Harmattan meets the north-easterly trade winds over
Africa.
C) The zone where the west winds meet the subtropical high pressure belt.
D) The zone where cold fronts form in the tropics.

1364. What is the name of the wind or airmass which gives to the main part of India its greatest
proportion of precipitation?
A) South-west monsoon.
B) South-east trade wind.
C) Indian, maritime tropical air mass.
D) Winter monsoon.

1365. In the areas of the ITCZ why are the heights of the tropopause not reported?
A) Because it is in the stratosphere.
B) Because it's too cold.
C) Because it is likely to be above your FL.
D) Because it cannot be measured.

1366. ITCZ weather is:


A) usually clear Wx, especially during the winter.
B) light winds from the south-west.
C) occasional showers with continuous rain.
D) often thundery through strong convergence.

1367. What weather is prevalent in the zone of easterly waves?


A) Frontal weather.
B) Clear skies.
C) Continuous rain.
D) Thunderstorms and rain.
1369. When are the rainy seasons in equatorial Africa?
A) December to February and July to October.
B) March to May and August to October.
C) April to July and December to February.
D) March to May and October to November

1371. What is the easterly wave?


A) A wave in the trade wind belt, moving from east to west, with severe convective
activity in the rear of its trough.
B) A wave in the trade wind belt, moving to the east, with severe convective action
in the rear of its trough.
C) A wave in the moderate latitudes, moving from east to west, with mostly stable
air in the rear of its trough.
D) An orographic-induced wave, moving to the east, in moderate latitudes.

1372. Flying from Marseilles to Dakar in summer where is the ITCZ?


A) Canaries.
B) Gibraltar.
C) Algeria.
D) Near Dakar.

1373. Trade winds are most prominent or strongest:


A) Lower troposphere over land.
B) Upper troposphere over land.
C) Upper troposphere over sea.
D) Lower troposphere over ocean.

1374. In which month does the humid monsoon in India start?


A) In June.
B) In December.
C) In October.
D) In March.

1375. The ITCZ affects:


A) The entire African continent.
B) West Africa between 10° and 30° N, and the north coasts of the Arabian Sea.
C) West Africa between 10° and 30° N only.
D) East Africa between 10° and 20° N.

1376. The low bringing tropical continental air masses to Europe in Summer is positioned in...
A) southern Italy.
B) Balkans.
C) southern France.
D) Azores.
1377. From which direction do the trade winds blow, in the southern hemisphere?
A) N
B) NE
C) SW
D) SE

1379. The Equatorial African rains occur...


A) March to May and October to November.
B) March to May and August to October.
C) June to August.
D) December to April.

1380. The Intertropical convergence zone (ITCZ) particularly affects:


A) western Africa, where it is situated between the 10° N and 30° N parallels, depending on
the time of the year.
B) western Africa between 10° and 20° N and the northern coasts of the Arabian sea in
July.
C) the Atlantic ocean, between latitudes 10° N and 30° N, depending on the time of year.
D) western Africa, at a latitude of 25° N in July.

1382. Tropical revolving storms that originate in the eastern Pacific, north of the equator, are called
..., and do their maximum damage...
A) hurricanes; on coasting in.
B) typhoons; well inland.
C) hurricanes; well inland.
D) cyclones; over the sea.

1383. On a flight from London to New York in summer, where would you cross the ITCZ?
A) You wouldn’ t.
B) Azores.
C) Newfoundland, Grand Banks.
D) New York.

1384. The ITCZ in July is?


A) passing through Freetown.
B) 20 deg N over west Africa.
C) over West Africa at 25 deg N and stretches up to the north of the Arabian sea.
D) near the Canaries.

1385. Which is the greatest region of tropical revolving storms?


A) The north-west Pacific, affecting Japan, Formosa, Korea and the Chinese coastline.
B) The Gulf of Mexico, affecting Texas, Louisiana and Florida.
C) The Bermuda Triangle.
D) The Caribbean Sea, affecting Florida and the Bahamas.
1386. What height is the tropopause and at what temperature?
A) At the equator 18km and -76 ° C.
B) At the poles 8km and -16 ° C.
C) At the equator 8km and -40 ° C.
D) At the pole 18km and -75 ° C.

1387. To build a hurricane, the following conditions, among others, must be fulfilled:
A) The surface temperature at least +27° C, and the building area 8 - 15° away from the equator.
B) Building area in the equatorial area.
C) Building area 5 - 6° away from the equator
D) The hurricane must be in the western areas of the ocean.

1388. What winds are mainly associated with the winter monsoon in the monsoon regions of
the Indian sub-continent?
A) South-easterly winds carrying warm and humid air.
B) North-easterly winds bringing dry and hazy air.
C) North-westerly winds bringing dry and hazy air.
D) South-westerly winds carrying warm and humid air.

1389. Monsoons are seasonal winds which:


A) develop due to high pressure over continents in winter and low pressure over continents in summer.
B) are from the SE over India in summer.
C) are never in combination with Trade Winds.
D) blow only in Asia.

1390. In January, mean high pressure areas are usually present over:
A) the South Pacific, the Azores, Australia.
B) the Aleutians, Australia, South America.
C) the Azores, the Aleutians, the South Pacific.
D) Greenland, Siberia, North West Australia.

1391. Which of the following statements about the ITCZ is correct?


A) It is normally fed with converging northern and southern trade winds.
B) Its furthest displacement from the Equator is normally about 45S.
C) Its associated weather is invariably strong convergence and heavy cumuliform cloud.
D) In the southern summer it is normally positioned entirely south of the Equator.

1392. Which of the following statements concerning trade winds is correct?


A) They occur only in the lower part of the troposphere and are more pronounced over the continents.
B) They reach up to the tropopause and are more pronounced over the oceans.
C) They reach up to the tropopause and are more pronounced over the continents.
D) They occur only in the lower part of the troposphere and more pronounced over the
oceans.

1393. Which of the following is true?


A) The monsoon low of India causes the SE trades which have crossed the Equator to form
the SW monsoon to turn left in the Bay of Bengal to affect the coast of India.
B) In winter the " Crachin" affecting Hong Kong is due to the SE monsoon.
C) The NE monsoon is due to the inflow of air into the large low pressure area of India.
D) Only the SW monsoon affecting the west coast of India is very moist.
Typical weather situations in mid-latitudes:
1396. What is a cold pool?
A) Cold air brought down from the arctic behind fast moving frontal systems
crossing the Atlantic west to east.
B) Air from Polar maritime origin.
C) Air from tropical continental origin.
D) Cold air found on the lee side of the Alps in winter in a cold north
westerly airstream.

1398. What kind of weather conditions do you encounter in a cold air pool centre?
A) Mostly clear skies.
B) Light rain.
C) Light drizzle.
D) Showers and thunderstorms.

1399. Which of the following statements concerning the Intertropical convergence zone is true?
A) It does not change its position over the oceans during the year.
B) There are frequent occurrences of CB.
C) It lies totally in the northern hemisphere in July and totally in the
southern hemisphere in January.
D) It is an area of low pressure and low relative humidity.

1400. What pressure systems affect the North Atlantic in summer?


A) Azores low, North Canadian low.
B) North Canadian low, Azores High.
C) Azores high, Scandinavian High.
D) Azores low, Scandinavian high.

1402. With a uniform pressure pattern and no thunderstorms around, what will the indication of the
aneroid altimeter of an aircraft parked on the ground do over a period of about ten minutes?
A) Increase rapidly.
B) Show strong fluctuations.
C) Decrease rapidly.
D) Apparently nothing, because any changes would be small.

1403. The greatest annual range of temperature will be found:


A) over large oceanic areas.
B) in polar regions.
C) over large land masses
D) in equatorial regions.
1407. What kind of weather system might you typically find between the 45 deg - 70 deg N?
A) Arctic front depression.
B) Polar high.
C) Polar front depression
D) Sub tropical high.

1408. A cold pool:


A) develops usually in winter when very unstable maritime polar or maritime
arctic air currents stream southwards along the eastern side of an
extensive ridge of high pressure, in association with occluded systems.
B) occurs frequently in winter to the south of the Alps when this region is
under the influence of cold north- westerly airstream.
C) normally disappears at night and occurs almost exclusively in summer.
D) is usually most evident in the circulation and temperature fields of the
middle troposphere and may show little or no sign on a surface chart.

1409. Which of the following statements is correct?


A) The outflow of air from the Siberian high over northern China an Japan is
initially north-westerly.
B) In winter the Savannah climatic region is governed by the equatorial
rains.
C) The easterly jet stream normally appears at the 200 mb level.
D) Trade wind seldom extend much above 5.000 feet.

1412. Which of the following weather conditions can be expected on the fringes of an advected cold
pool when a suitable trigger action is present:
A) a cold front with Cu, Cb and showers.
B) a warm front with Ns, St and drizzle.
C) anticyclonic circulation and stratus.
D) TRS conditions.

1413. What is a cold pool?


A) Found south of the Alps if there is NW airflow.
B) Cold pool is most evident behind polar frontal weather in mid temperate
areas with little or no sign on significant weather charts.
C) Air trapped on the leeward side of mountain ranges.
D) Cool area of weather which disappears at night.
Local seasonal weather and wind:
1414. What is the flight hazard associated with the Harmattan?
A) Wind shear.
B) Sand up to FL150.
C) Dense fog.
D) Dust and poor visibility sometimes down to within fog limits.

1415. Sub-tropical jets in the northern hemisphere will be found at latitudes... and in the southern
hemisphere at latitudes...
A) 10N to 15N; 25S to 30S
B) 25N to 30N; 15S to 20S
C) 45N to 60N; 40S to 45S
D) 25N to 40N; 30S to 35S

1416. When are cyclones most likely? (Northern Hemisphere)


A) Late Autumn.
B) Mid Winter.
C) Late Spring.
D) Late Summer.

1417. What is the Bora?


A) Cold katabatic wind over the Adriatic.
B) Warm anabatic wind blowing to the Mediterranean.
C) Northerly wind blowing from the Mediterranean.
D) An anabatic wind in the Rockies.

1419. The general weather conditions on the east coast of Malaya in January would be:
A) Cb and thunderstorms moving in on the SW wind.
B) Cu and Cb and thunderstorms developing during the day.
C) fine weather and mainly clear skies.
D) deep layers of Ns and continuous moderate rain.

1420. The Foehn wind is a:


A) warm fall wind.
B) cold anabatic wind.
C) warm anabatic wind.
D) cold fall wind.

1421. At what times of the year does the length of the hours of daylight change most rapidly?
A) Spring Equinox and Summer Solstice.
B) Autumn Equinox and Winter Solstice.
C) Summer Solstice and Winter Solstice.
D) Spring Equinox and Autumn Equinox.
1422. If flying in the Alps with a Foehn effect from the south...
A) clouds will be covering the southern passes of the Alps.
B) wind backing and gusting on the northern side.
C) CAT on the northern side.
D) convective weather on the southern passes of the Alps.

1423. Define the Chinook:


A) Warm and dry air descending at the leeward side of the Rocky Mountains.
B) Warm and dry air ascending on the West Side of the Rocky Mountains.
C) Cold and dry air descending at the leeward side of the Appalachian Mountains.
D) A dust and sand laden north-easterly wind in Northwest Africa.

1424. A strong, dry and warm katabatic wind, produced by prior enforced ascent of air over hills or
mountains is known as a:
A) Harmattan.
B) Mistral.
C) Foehn.
D) Bora.

1425. Which one of the following local winds is a Foehn wind?


A) Bora.
B) Sirocco.
C) Chinook.
D) Harmattan.

1427. Equatorial easterly jets occur:


A) 17N in the Northern Hemisphere summer.
B) 17S in the Southern Hemisphere summer.
C) the Southern Hemisphere all year.
D) the Northern Hemisphere all year.

1428. x8/8 stratus base 200 FT/AGL is observed at sunrise at an aerodrome in the north of France;
the QNH is 1028 hPa and there is a variable wind of 3 kt. What change in these clouds is likely at
12:00 UTC in summer and winter?
A) Winter: OVC base 500 FT/AGL; summer SCT base 3.000 FT/AGL.
B) Winter: BKN base 2.500 FT/AGL; summer BKN base 3.500 FT/AGL.
C) Winter: SCT base 3.000 FT/AGL; summer OVC base 500 FT/AGL.
D) Winter: clear sky; summer BKN CB base 1.500 FT/AGL.

1429. What is the name of the northerly, cold and strong wind, that sometimes blows over a certain
part of Europe?
A) Mistral.
B) Bora.
C) Typhoon.
D) Foehn.
1430. The Chinook is a:
A) down slope wind that occurs particularly at night as air cools along
mountain slopes.
B) warm and dry wind that forms as air descends on the leeward side of the
Rocky Mountains.
C) warm anabatic wind up the slopes of snowfields or glaciers.
D) very cold wind with blowing snow.

1431. Considering the route indicates from Lisbon to Freetown, the Harmattan is a:
A) NE wind affecting north-west Africa during November to April reducing
visibility in rising dust.
B) warm southerly dust-bearing wind affecting the coast of North Africa.
C) localised depression giving squally winds.
D) SW monsoonal wind causing extensive areas of advection fog along the
West African coast south of 15° N.

1432. For an aircraft what are the meteorological dangers associated with a Harmattan wind?
A) Thunderstorms.
B) Hail.
C) Sand up to FL 150.
D) Dust and poor visibility.

1433. Which of the following is an example of a Foehn wind?


A) Ghibli.
B) Chinook.
C) Bora.
D) Harmattan.

1434. Flying form London to Bombay in January, what average wind might you expect?
A) Light westerly.
B) Light easterly.
C) Westerly polar front jet stream.
D) Tropical Easterly jet.

1436. Which of the following zones is most likely to encounter little or no precipitation?
A) Frontal zones.
B) The north side of the alps with a prevailing Foehn from the south.
C) Occlusions.
D) The north side of the alps with a prevailing Foehn from the north.
1437. What is the name of the hot, local wind, that blows downwards from mountain chains? In the
Alps, for instance, it may exist both as a southerly or northerly wind depending on the weather
situation.
A) Bora.
B) Mistral.
C) Foehn.
D) Sirocco.

1439. Which of the following statements about the West African Tornado (WAT) is correct?
A) the WAT is another name for TRSs off the west African coast.
B) the WAT is the E/W line of thunderstorms on the ITCZ.
C) the WAT is a line squall caused by atmospheric waves.
D) the WAT is similar to the North American tornado.

1440. Flying from an area of low pressure in the SH at low altitudes, where is the wind coming from?
A) Right and slightly on the nose.
B) Left and slightly on the nose.
C) Right and slightly on the tail.
D) Left and slightly on the tail.

1441. Which weather phenomena are typical for the northern side of the Alps with stormy winds from
the south (Foehn)?
A) Good visibility, turbulence.
B) Continuous precipitation, severe turbulence.
C) Decrease in temperature, moderate to severe icing.
D) Icing, huge mass of clouds

1442. What typical weather do you encounter during Foehn conditions north of the Alps?
A) Icing.
B) Good weather.
C) Turbulence during south Foehn.
D) Heavy rain and NS clouds.

1443. The Bora is a:


A) cold katabatic wind with the possibility of violent gusts.
B) cold katabatic wind with gusts associated with a maritime air mass.
C) squally warm katabatic wind which occurs mainly in summer.
D) cold katabatic wind always associated with clouds and heavy showers.

1444. The Sirocco occurs:


A) ahead of a frontal depression moving east along the coast.
B) due to intense heating causing convectional dust storms.
C) due to the steep temperature gradient along the N. African coast.
D) behind a frontal depression moving east along the coast.
1445. When travelling from Stockholm (55N 18E) to Rio de Janeiro (22S 80W), you encounter:
A) polar Front jetstream then sub tropical jet then polar jet.
B) one Sub tropical jetstream.
C) polar Front jet then 1 or 2 sub tropical jets.
D) one sub tropical jetstream then one polar front jet.
1446. In a Tropical Downpour the visibility is sometimes reduced to:
A) 500m.
B) 1000m.
C) less than 100m.
D) 200m.
1447. What weather conditions in the region of the Alps would you expect with Foehn from south?
A) Heavy clear air turbulence on the southern side of the Alps.
B) Strong north winds on the southern side of the Alps.
C) Clouds, on the southern sides of passes in the Alps.
D) Heavy airframe icing conditions on the northern side of the Alps.
1448. What is a dry, sand and dust-laden north-easterly wind in the winter in Northwest Africa?
A) Sirocco.
B) Harmattan.
C) Pampero.
D) Khamsin.
1449. The winds blowing over the Indian Ocean in July, North of the equator will be:
A) SW.
B) NW.
C) SE.
D) NE.
1452. Consider the following statements regarding local winds:
1. The Pampero is a Foehn type wind
2. The Mistral is a valley wind
3. The Sirocco and the Ghibli are similar winds which blow in North African coastal regions giving
poor visibility
4. The Bora blows ahead of depressions moving west to east in the eastern Mediterranean
Of these statements:
A) only 3 is correct.
B) 1, 2 and 3 are correct.
C) 3 and 4 are correct.
D) 2 and 3 are correct.
1453. Which of the following is a cold katabatic wind flowing towards the Adriatic Sea?
A) Sirocco.
B) Mistral.
C) Ghibli.
D) Bora.
1454. An example of a Foehn effect wind is the:
A) Sirocco.
B) Shamal.
C) Chinook.
D) Bora.

1455. Define the Foehn:


A) Strong, cold wind, mostly observed in the Rhone valley, blowing towards Southern France.
B) Warm and dry air ascending on the west side of the Alps.
C) Warm and dry air descending at the leeward side of the Alps.
D) Cold and dry air descending at the leeward side of the Appalachian Mountains.

1456. How is the weather in Bombay in early July influenced?


A) By the trade winds.
B) By NE monsoon.
C) By the Harmattan.
D) By SW monsoon.

1457. What is characteristic of the Pampero?


A) Katabatic winds in the Atlas Mountains.
B) A marked advance of cold air in South America.
C) Foehn conditions in the Spanish Pyrenees.
D) A marked advance of cold arctic air in North America.

1458. What time of year is the tornado season in N. America?


A) Early Summer.
B) Early Spring.
C) Summer and early Autumn.
D) Spring and early Summer.

1459. A Foehn wind occurs on the:


A) windward side of a mountain range and is caused by surface heating.
B) leeward side of a mountain range and is caused by significant moisture loss by
precipitation from cloud.
C) windward side of a mountain range and is caused by surface cooling and reverse
air flow.
D) leeward side of a mountain range and is caused by the condensation level being
lower on the leeward side than on the windward side.

1460. Prevailing winds in North West. Africa (07 N Gulf of Guinea area) will be:
A) SE monsoon in summer, NW trade winds in winter.
B) SE trade wind in summer, NE monsoon in winter.
C) SW monsoon in summer, NE trade winds in winter.
D) SE trade wind in winter, NE monsoon in summer.
1462. What is the strong relatively cold katabatic wind, blowing down the northern Adriatic coast,
mainly during the winter and spring called?
A) Sirocco.
B) Mistral.
C) Bora.
D) Ghibli.

1464. What is the reason for seasonal changes in climate?


A) Because of the difference between the Tropical Year and the Calendar Year.
B) Because the Earths spin axis is inclined to the plane of its orbit round the Sun.
C) Because the Earth's orbital speed round the Sun varies according to the time of
the year.
D) Because the distance between the Earth and the Sun varies over a year.

1465. Local winds can affect the Mediterranean in winter. Which of the following lists of three is
correct?
A) Mistral; Pampero; bora.
B) Bora; mistral; sirocco.
C) Shamal; sirocco; bora.
D) Simoom; Khamsin; shamal.

1467. Consider a route from Madrid (40º N) to Lagos, West Africa (5º N). What upper winds would you
expect in August from 20º N to Lagos?
A) Easterly winds.
B) Westerly winds.
C) Westerly then easterly winds.
D) Easterly then westerly winds

1468. What are the characteristics of the Bora?


A) It is a cold and very strong wind that blows mainly in winter from a
tableland downwards to the Adriatic.
B) It is a warm and moist, south-westerly wind experienced in the eastern
Mediterranean, that usually carries precipitation.
C) It is a very cold wind that blows mainly in winter from a north-westerly
direction in the Mediterranean.
D) It is a dry and hot southerly wind experienced in the Sahara desert, that
often carries dust.

1469. A dry, sand- and dust-laden North Easterly wind that blows in winter over large parts of North
West Africa is known as a:
A) Sirocco.
B) Pampero.
C) Harmattan.
D) Khamsin.
1472. What is the name of the dry, dusty wind blowing in North West Africa from the North East?
A) Harmattan.
B) Ghibli.
C) Khamsin.
D) Pampero.

1475. Which of the following statements is correct?


A) In winter the Savannah climatic region is governed by the equatorial
rains.
B) The outflow of air from the Siberian high over northern China and Japan
is initially north-westerly.
C) Trade winds seldom extend much above 5000ft.
D) The easterly jet stream normally appears at the 200mb level.

1476. What type of jet stream blows constantly through the northern hemisphere?
A) Polar front.
B) Arctic.
C) Equatorial.
D) Sub tropical.
1477. In July in the northern hemisphere, the relationship between surface type, surface temperature
and surface pressure is best described as:
A) land - low temperature - high pressure.
B) sea - high temperature - high pressure.
C) sea - low temperature - low pressure.
D) land - high temperature - low pressure.

1478. The Bora is a...


A) wind with a airmass of maritime origin.
B) warm squally katabatic wind.
C) wind with a airmass of arctic origin.
D) wind that may produce violent gusts.
Flight Hazards
Icing:
1481. Which of the following statements is true regarding moderate to severe airframe icing?
A) It always occurs in altostratus cloud.
B) It is likely to occur in nimbostratus cloud.
C) It will occur in clear-sky conditions.
D) It may occur in the uppermost levels of a cumulonimbus capillatus
formation.

1482. Hoar frost forms on an aircraft as a result of:


A) water vapour turning directly into ice crystals on the aircraft surface.
B) freezing rain striking the aircraft.
C) droplets forming on the aircraft and then freezing.
D) small super-cooled droplets striking the aircraft.

1483. The situation where rain ice is likely to occur must be avoided. This would be when flying:
A) ahead of a warm front with the OAT gauge reading -5C.
B) through an active cold front.
C) in a polar maritime airstream behind a cold front.
D) in the warm sector of a frontal depression during winter.

1484. At what airframe temperature would you expect the worst icing effects:
A) -20 degrees Celsius.
B) -15 degrees Celsius.
C) -5 degrees Celsius.
D) -30 degrees Celsius.

1485. At what degree of icing should ICAOs Change course and/or altitude immediately instruction be
followed?
A) Moderate.
B) Light.
C) Extreme.
D) Severe.

1486. In which of these cloud types can icing be virtually ruled out?
A) CU
B) SC
C) CI
D) NS
1487. Which of the following, without orographic intensification, will give rise to light to moderate icing
conditions?
A) Ns and Cc
B) Ns and Cs
C) As and Ac
D) Cb and Ns

1488. Rime ice is caused by:


A) large Supercooled water droplets.
B) rapid re-freezing of large water droplets.
C) small Supercooled water droplets.
D) slow freezing of water droplets onto the wing.

1489. Why is clear ice such a problem?


A) Not translucent and forms along leading edges.
B) Very heavy and can affect aircraft controls and surfaces.
C) Translucent and forms along leading edges.
D) Forms in clear air.

1490. In which of these temperature bands is ice most likely to form on the aircrafts surface?
A) +10° C to 0° C.
B) 0° C to -10° C.
C) -35° C to -50° C.
D) -20° C to -35° C.

1491. If you encounter freezing rain, do you:


A) climb to the warmer air above.
B) climb to the cooler air above.
C) accelerate.
D) descend.

1492. Which situation most frequently gives freezing rain over Central Europe?
A) Warm front.
B) Cold front.
C) Cold occlusion.
D) Warm occlusion.

1493. What cloud type are you least likely to get icing from?
A) Ci
B) Cu
C) Ns
D) St
1494. In what cloud is icing and turbulence most severe?
A) Cb
B) Ns
C) Sc
D) Ci

1495. Which of the following clouds would present the greatest danger of icing:
A) Ns
B) Ts
C) Sc
D) Cs

1496. The icing in cloud which forms over hills is likely to be more severe than in the same type of
cloud over level terrain because:
A) enforced ascent of air releases more water, which is retained in the cloud
by the increased upward components.
B) orographic lifting causes the freezing level to rise and increases the free
water content of the cloud.
C) increases the temperature inside the cloud by forcing the release of latent
heat so causing the air to hold more water vapour.
D) adiabatic cooling lowers the freezing level and the water content of the
cloud.

1497. You will get least amount of icing in which cloud?


A) SC
B) CS
C) NS
D) AS

1498. A winter day in northern Europe with a thick layer of stratocumulus clouds and temperature
close to zero degrees C at ground level, you can expect:
A) A high probability for icing in clouds. Severe icing in the upper part due to
accumulation of large droplets.
B) Turbulence due to a strong inversion, but no icing because clouds consist
of ice crystals.
C) Decreasing visibility due to snowfall below cloud base, but only light icing
in clouds.
D) Reduced visibility and light icing in clouds.

1499. A small supercooled cloud droplet that collides with an airfoil will most likely:
A) freeze immediately and create clear ice.
B) travel back over the wing, creating rime ice.
C) travel back over the wing, creating clear ice.
D) freeze immediately and create rime ice.
1500. Which of the following conditions is most likely to cause airframe icing?
A) +FZRA
B) PE
C) SHSN
D) GR

1501. Icing and raindrops on the wings of an aircraft normally causes:


A) the stalling speed to increase.
B) the interference drag to decrease.
C) the stalling speed to decrease.
D) the interference drag to increase.

1502. A thin coating of hoar frost on the airfoil surface:


A) does not affect take-off performance.
B) affects the aspect ratio of the wing.
C) has deleterious effects on the lift of the wing.
D) only affects stability.

1503. Which airframe icing is the most dangerous?


A) Hoar frost.
B) Rime ice.
C) Clear ice.
D) Mixed ice.

1504. At what degree of icing can ICAO (No change of course and altitude necessary) recommendation
be followed?
A) Severe.
B) Extreme.
C) Moderate.
D) Light.

1505. Which of the following statements is true:


A) the frost point can be reached when the saturation vapour pressure
reaches 0° C.
B) a frost point can only exist when the dew point is below 0° C.
C) air cooled below its dew point will eventually reach its frost point.
D) the frost point can be achieved only with super-saturated air.

1506. Which one of the following precipitation types gives the most severe icing?
A) Freezing rain.
B) Ice pellets.
C) Mixed rain and snow.
D) Snowfall.
1507. Hoar frost is most likely to form when:
A) taking off from an airfield with a significant ground inversion.
B) flying inside Stratiform clouds.
C) flying in supercooled drizzle.
D) flying inside convective clouds.

1508. Which of the following statements is true regarding moderate-to-severe airframe icing?
A) It always occurs in altostratus cloud.
B) It may occur in the uppermost levels of a cumulonimbus capillatus
formation.
C) It is unlikely to occur in nimbostratus cloud.
D) It will not occur in clear-sky conditions.

1509. Clear ice forms on an aircraft by the freezing of:


A) small supercooled water drops.
B) snow.
C) water vapour.
D) large supercooled water drops.

1510. The possibility of significant icing at altitude, should be expected when on ground you observe:
A) hail.
B) corn-snow.
C) ice grains.
D) snow.

1511. You would expect mixed clear and rime icing in the temperature range... and no icing below...
A) -25º C to -45º C; -55º C
B) -25º C to -45º C; -45º C
C) -3º C to -25º C; -45º C
D) -3º C to -45º C; -55º C

1512. Which of the following cloud types are most likely to produce light to moderate icing when they
are not subject to orographic lifting and consist of supercooled cloud droplets?
A) Stratus and cumulonimbus.
B) Stratocumulus and cirrostratus
C) Altocumulus and altostratus.
D) Altostratus and cirrocumulus.

1513.In which of these cloud types can icing be virtually ruled out?
A) AS
B) CS
C) NS
D) SC
1514. Which of the following has the greatest effect upon ice accretion?
A) Size of droplet and temperature.
B) Relative humidity and temperature.
C) Freezing levels.
D) Speed and shape of aerofoil.
1515. Which one of the following statements concerning the formation of aircraft icing is most correct?
A) Probability of icing increases when dry snow starts to fall from a cloud.
B) A cloud consisting of both supercooled cloud droplets and ice crystals
produces aircraft icing
C) Greatest risk of icing conditions is experienced in cirrus clouds.
D) Risk for icing increases when cloud temperature decreases well below
minus 12 degrees C.
1516. What type of icing requires immediate diversion?
A) Extreme.
B) Moderate.
C) Light.
D) Severe.
1517. Freezing fog exists if fog droplets:
A) freeze when temperature falls below zero.
B) are frozen.
C) are supercooled.
D) are freezing very rapidly.
1518. Flying in freezing rain a hard, partly transparent layer of ice rapidly forms on the aircraft and in
response the pilot intends to make an immediate change of altitude and/or heading. What do we call
this type of icing and how is it classified according to ICAO?
A) Clear ice/coating of clear ice severe icing
B) Rime ice Moderate icing.
C) Clear ice Light icing.
D) Frost Severe icing.
1519. Under which conditions would you expect the heaviest clear ice accretion to occur in a CB?
A) Between -30° C and -40° C.
B) Close to the freezing level.
C) Between -2° C and -15° C.
D) Between -20° C and -30° C.
1520. In which meteorological environment is aircraft structural icing most likely to have the highest
rate of accumulation?
A) Freezing rain.
B) High humidity and freezing temperature.
C) Heavy wet snow.
D) Cumulonimbus clouds.
1521. Glaze or clear ice is formed when supercooled droplets are:
A) small and freeze rapidly.
B) large and at a temperature just below freezing.
C) small and at a temperature just below freezing.
D) of any size at temperatures below -35° C.

1522. Airframe icing:


A) Cannot occur outside cloud or precipitation.
B) Can only occur if the Outside Air Temperature (OAT) is below freezing.
C) Can occur outside cloud or precipitation.
D) Can only occur in side cloud or precipitation if the Outside Air
Temperature (OAT) is below freezing.

1523. Aircraft with thick wing (T) and thin wing (S) fly at the same TAS and altitude through cloud
containing small supercooled water droplets. What extent of icing will be experienced?
A) T more, S less.
B) S and T same icing.
C) Nothing as its only super cooled water droplets.
D) S more, T less.

1524. In which of the following situations is an aircraft most susceptible to icing?


A) Level flight in snowfall below a nimbostratus layer.
B) Flying in dense cirrus clouds.
C) Level flight below a rain producing cloud when OAT is below zero degrees C.
D) Flying in heavy drizzle.

1525. How does a pilot react to heavy freezing rain at 2000 FT/AGL, when he is unable to deice, nor
land?
A) He descends to the warm air layer below.
B) He turns back before the aircraft loses manoeuvrability.
C) He continues to fly at the same altitude.
D) He ascends to the cold air layer above.

1526. One of the most serious consequences of icing on the wings of an aircraft is?
A) The stalling speed increasing substantially.
B) The lift coefficient increasing as the aircraft becomes heavier.
C) The wing construction being unable to bear the increased load.
D) Lift becoming too great as the aircraft becomes heavier.

1527. Carburettor icing may be expected to occur in clear air when the relative humidity is ..., the
temperature is ... and with ... power set.
A) 20%; 30° C; climb
B) 40%; +20° C; descent
C) 30%; +30° C; descent
1528.A vertical temperature profile indicates the possibility of severe icing when the temperature
profile:
A) indicates temperatures above 3° C.
B) indicates temperatures below -40° C.
C) coincides with a dry adiabatic lapse rate.
D) intersects the 0° C isotherm twice.

1529. If flying en-route and you encounter severe icing, according to ICAO recommendations, you
should...
A) divert.
B) climb below the clouds.
C) climb above the clouds.
D) continue.

1530. Aircraft A has a sharp leading edge and a thin aerofoil. Aircraft B has a thick cambered wing
aerofoil. If they are flying at the same TAS into clouds with small supercooled water droplets then:
A) icing depends upon the differential kinetic heating.
B) B gets more icing than A.
C) both get the same.
D) A gets more icing than B.

1531. In which cloud type does moderate to severe airframe icing most probably occur?
A) AC
B) CS
C) ST
D) NS

1532. On the approach, the surface temperature is given as -5° C. The freezing level is at 3000
FT/AGL. At 4000 FT/AGL, there is a solid cloud layer from which rain is falling. According to the
weather briefing, the clouds are due to an inversion caused by warm air sliding up and over an
inclined front. Would you expect icing?
A) Yes, between ground level and 3000 FT/AGL.
B) No, absolutely no icing will occur.
C) No, flights clear of cloud experience no icing.
D) Yes, but only between 3000 and 4000 FT/AGL.

1533. During the formation of rime ice in flight, water droplets freeze:
A) rapidly and do not spread out.
B) rapidly and spread out.
C) slowly and spread out.
D) slowly and do not spread out.

1534. Rime ice forms through the freezing onto aircraft surfaces of:
A) large supercooled water drops.
B) snow.
C) small supercooled water drops.
D) water vapour.
1535. Small supercooled water droplets hit the aerofoil, will it:
A) freeze on impact giving clear ice.
B) partially freezing and running back giving rime ice.
C) freeze on impact giving rime ice.
D) partially freezing and running back giving clear ice.
1536. Carburettor icing is unlikely:
A) In clouds.
B) At temperatures between -10° C and -30° C.
C) At temperatures above freezing.
D) When the relative humidity is 40%.
1537. In which of the following situation is carburettor icing most likely to be serious over Northwest
Europe?
A) Winter, cold air mass, descent power.
B) Winter, cold air mass, takeoff power.
C) Summer, warm air mass, cruise power.
D) Summer, warm air mass, descent power.
1538. Which one of the following statements concerning icing in freezing rain is correct?
A) Clear ice forms when large water drops freeze.
B) Rime ice forms when large water drops freeze.
C) Hoar frost form when small water droplets freeze.
D) A coating of clear ice forms when water vapour sublimates.
1539. Rain ice may be found in:
A) D2.
B) B3.
C) D7.
D) C9.

1540. Which of the following, without orographic intensification, will give rise to light to moderate icing
conditions?
A) It will occur in clear-sky conditions.
B) It always occurs in altostratus cloud.
C) It is likely to occur in nimbostratus cloud.
D) It may occur in the uppermost levels of a cumulonimbus capillatus formation.

1541. Which of the following temperature conditions is worst for icing?


A) -2 deg C to -15 deg C.
B) near the freezing level.
C) -25 deg C to -30 deg C.
D) -15 deg C to -20 deg C.
1542. Atmospheric soundings give the following temperature profile: 3000 FT 15° C 6000 FT 8° C
10000 FT 1° C 14000 FT -6° C 18000 FT -14° C 24000 FT -26° C At which of the following flight levels
is the risk for aircraft icing, in cloud, greatest?
A) FL 150
B) FL 80
C) FL 220
D) FL 180

1543. Clear ice forms as a result of:


A) supercooled water droplets spreading during the freezing process.
B) ice pellets splattering on the aircraft.
C) supercooled droplets freezing on impact.
D) water vapour freezing to the aircraft.

1544. Flying over an airfield, at the surface the temp. is -5 C, freezing level is at 3000ft, rain is falling
from clouds with a base of 4000ft caused by warm air rising above cold air. Where would you
experience icing?
A) Never.
B) Below 3000ft.
C) Between 3000 - 4000ft.
D) No icing because your not in cloud.

1545. Which of the following factors have the greatest effect on the formation of the various types of
ice on an aircraft?
A) Cloud temperature and droplet size.
B) Relative humidity inside the cloud.
C) Aircraft speed and curvature of the airfoil.
D) Aircraft speed and size of cloud droplets.

1546. Where are you most likely to find moderate to severe icing?
A) In upper levels of Cumulonimbus Capillatus.
B) Stratus.
C) Nimbostratus.
D) Cirrus.

1547. One in-flight condition necessary for structural icing to form is:
A) Stratiform clouds.
B) visible moisture.
C) small temperature/dewpoint spread.
D) temperature below 35 degrees Celsius.

1548. Clear ice is dangerous because it:


A) is heavy and is difficult to remove from the aircraft surfaces.
B) is not translucent and forms at the leading edges.
C) spreads out and contains many air particles.
D) is translucent and only forms at the leading edges.
1549. Which of these statements about icing is correct?
A) Large amounts of icing if temperature is way below -12 deg C.
B) Ice will occur going through cirrus cloud.
C) Icing increases if dry snow starts to fall from cloud.
D) Icing will occur if super-cooled water and ice are present.

1550. The type of icing that occurs in dense clouds with large supercooled drops that have a
temperature of -5° C is most likely to be:
A) cloudy ice.
B) hoar frost.
C) clear ice.
D) rime ice.

1551. In which environment is aircraft structural ice most likely to have the highest rate of accretion?
A) Snow.
B) Cirrus clouds.
C) Stratus clouds.
D) Freezing rain.

1552. The worst combination of a high rate of accretion of clear ice may be found:
A) in Cb in Europe at FL150.
B) in Cb in India at FL160.
C) in Cb in Europe at FL250.
D) in Cb in India at FL50.

1553. What is true regarding super cooled water droplets?


A) Always below -60 deg C.
B) All large.
C) All below 0 deg C.
D) All small.

1554. What is the most severe form of icing?


A) Rime ice.
B) Clear ice.
C) Dry Ice.
D) Hoar frost.

1555. ICAO statement - no diversion necessary, de-icing is not required or is effective. The icing in
this case is...
A) severe.
B) extreme.
C) moderate.
D) light.
1556. The most dangerous form of airframe icing is:
A) rime ice.
B) clear ice.
C) dry ice.
D) hoar frost.

1557. At what degree of icing should ICAOs Change of course and/or altitude desirable
recommendation be followed?
A) Extreme.
B) Severe.
C) Light.
D) Moderate.

1558. Where is icing worst?


A) -16 deg C to -30 deg C etc.
B) Near freezing level.
C) -2 deg C to -15 deg C
D) Near condensation level.

1559. You are in a descent with the throttle closed. Which of the following conditions presents the
most serious danger from piston engine intake icing?
A) OAT : +25º C; Relative humidity :80%.
B) OAT : +15º C; Relative humidity :60%.
C) OAT : +5º C; Relative humidity :60%.
D) OAT : +25º C; Relative humidity :40%.

1560. Large supercooled water drops, which freeze on impact on an airplane, form:
A) rime ice.
B) clear ice.
C) hoar frost.
D) cloudy ice.

1561. While descending through a cloud cover at high level, a small amount of a white and rough
powder like contamination is detected along the leading edge of the wing. This contamination is
called:
A) Clear ice.
B) Rime ice.
C) Mixed ice.
D) Frost.

1562. Which of the following conditions is most favourable for the formation of carburettor icing if an
aircraft is descending with glide power?
A) Relative Humidity 40%, ambient temperature + 30° C.
B) Relative Humidity 30%, ambient temperature -10° C.
C) Relative Humidity 30%, ambient temperature +10° C.
D) Relative Humidity 25%, ambient temperature +25° C.
1563. You have been flying for some time in dense layered cloud. The outside air temperature is -25°
C. Which of the following statements is true?
A) Severe airframe icing is unlikely under these conditions.
B) If you do not have weather radar on board there is no need to worry, as
CB is unlikely to form in such cloud.
C) In a dense layered cloud icing is unlikely also at an outside air
temperature of -5° C.
D) Severe airframe icing is quite likely under these conditions.

1564.Two aircraft, one with a sharp wing profile (S), and the other with a thick profile (T), are flying
through the same cloud with same true airspeed. The cloud consists of small supercooled droplets.
Which of the following statements is most correct concerning ice accretion?
A) Neither of the aircraft accumulate ice due to the small size of droplets.
B) Aircraft T experiences more icing than S.
C) Aircraft S and T experience the same amount of icing.
D) Aircraft S experiences more icing than T.

1565. Clear ice is most likely to form when the ambient air temperature is...
A) -20 deg C to -40 deg C.
B) -20 deg C to -30 deg C.
C) -10 deg C to -17 deg C.
D) -40 deg C to -60 deg C.
Turbulence:
1567. Fair weather cumulus gives an indication of:
A) smooth flying below.
B) turbulence.
C) poor visibility.
D) thunderstorms.

1568. From the pre-flight briefing you know a jet stream is at 31,000 ft whilst you are at FL270. You
experience moderate CAT, what would be the best course of action?
A) Stay level.
B) Descend.
C) Climb.
D) Reduce speed.

1569. How does moderate turbulence affect an aircraft?


A) Changes in altitude or attitude occur but the aircraft remains in positive
control at all times.
B) Large, abrupt changes in altitude or attitude occur but the aircraft may
only be out of control momentarily.
C) Continued flight in this environment will result in structural damage.
D) Rapid and somewhat rhythmic bumpiness is experienced without
appreciable changes in altitude or attitude.

1570. When CAT (clear air turbulence) conditions are anticipated, the following procedure should be
used as soon as turbulent air penetration speed is established:
A) The attitude indicator should be used as secondary flight instrument.
B) The airspeed indicator should be used as primary flight instrument.
C) Maintain wings level and control pitch attitude smoothly.
D) Prepare the use of large control inputs to fight the excessive G-forces that
may occur.

1571. An aircraft is approaching a cold front from the warm air mass side at FL 270 and experiencing
moderate to severe turbulence. A jet stream is forecast to be at FL 310. The shortest way to get out of
this turbulence is by:
A) Climbing.
B) Maintain FL 270.
C) Turn right.
D) Descending.
1572. Clear air turbulence (CAT) should be anticipated when:
A) A sharp trough area aloft is present with wind speed considerably higher
than that of jetstream winds.
B) A sharp trough area aloft is present, even though the wind speeds may
be considerably less than that of jetstream winds.
C) Whenever a high pressure is present close to a thunderstorm area.
D) Immediately after the passage of a thunderstorm.

1573. Conditions favourable for low-level frictional turbulence are:


A) strong wind, smooth terrain, stable lapse rate.
B) light wind, rough terrain, stable lapse rate.
C) strong wind, rough terrain, steep lapse rate.
D) strong wind, rough terrain, stable lapse rate.

1574. One of the main hazards when flying at high altitude close to a jet stream is:
A) prolonged discomfort for passengers if flying in same direction as jet.
B) low groundspeed if flying in opposite direction to jet.
C) small buffet margin when encountering CAT.
D) CAT if flying directly below the core of the jet .

1575. In a polar front jet stream in the Northern Hemisphere, where is there likely to be the greatest
probability of turbulence?
A) Looking downstream, to the right.
B) In the core.
C) Above the jet core in the boundary between warm and cold air.
D) Looking downstream, to the left.

1576. How do you recognise high level jet streams and associated CAT?
A) Lenticularis.
B) High level dust.
C) Streaks of Cirrus.
D) High pressure centre at high level.

1577. Wake turbulence is created by:


A) behind high buildings less than 300 m from the runway.
B) wind forces greater than 50 kts.
C) the wingtips of heavy aircraft during takeoff and landing.
D) cumulonimbus clouds.

1578. Your flight forecast shows that there is an area of severe turbulence ahead. You could therefore
expect:
A) IAS fluctuating more than 25 KT, food service and walking impossible.
B) accelerometer showing 0.5 to 1.0g, unsecured objects dislodged.
C) momentary loss of control, occupants feeling definite strain against seat belts/shoulder. straps
D) IAS fluctuating between 15 & 25 KT, unsecured objects tossed about.
1579. Maximum clear air turbulence will be found at a jetstream ... the jet core, in the ... air, and on
the ... side of the jet.
A) above; cold; high pressure
B) below; cold; low pressure
C) below; cold; frontal
D) above; warm; cold air

1580. Possible severe turbulence will be encountered in mountain waves in:


A) lenticularis cloud.
B) on the windward side.
C) roll cloud.
D) at the crest of the mountain.

1581. All pilots encountering Clear Air Turbulence are requested to report it. You experience CAT
which causes passengers and crew to feel definite strain against their seat belt or shoulders straps.
Unsecured objects are dislodged. Food service and walking are difficult. This intensity of CAT should
be reported as:
A) moderate.
B) severe.
C) light.
D) extreme.

1582. Maximum turbulence associated with the standing waves is likely to be:
A) down the lee side of the ride and along the surface.
B) two wavelengths down wind and just above the surface.
C) just below the tropopause above the ridge.
D) approximately one wavelength down wind of, and approximately level
with, the top of the ridge.

1583. What is the usual procedure when encountering CAT en-route?


A) Turn around immediately.
B) Accelerate through it and stay level.
C) Request climb to get out of it.
D) Request descent immediately to clear it.

1584. The degree of CAT experienced by an aircraft is proportional to:


A) stability of the air.
B) height of the aircraft.
C) intensity of solar radiation.
D) intensity of vertical and horizontal wind shear.
1585.Turbulence inside thunderstorms is the result of the up and down movement of the air. Typical
values for updrafts and downdrafts in a mature thunderstorm are:
A) 3000-4000fpm; 4000-6000fpm.
B) 4000-6000fpm; 3000-4000fpm.
C) 10,000fpm; 6000fpm.
D) 3000fpm; 2000fpm.
1586. The Jetstream and associated clear air turbulence can sometimes be visually identified in flight
by:
A) long streaks of cirrus clouds.
B) a high-pressure centre at high level.
C) a constant outside air temperature.
D) dust or haze at high level.

1587. Moderate turbulence can be expected in...


A) Alto-cumulus lenticularis.
B) Stratus.
C) Nimbostratus.
D) Cirrocumulus.

1588. The turbulence which occurs at high flight levels (above FL 250) is mainly of the type Clear Air
Turbulence. In what way can moderate to severe clear air turbulence affect an aircraft, the flight and
the passengers?
A) The turbulence can be resembled with the roughness of a washing-board
(small scale) and will not have influence on the aircraft and its solidity,
but will make flight a little more difficult. The passengers will seldom
notice anything of this turbulence.
B) The turbulence is a small scale one and can cause damage of worn out
type. The manoeuvring of the aircraft will be made more difficult or even
impossible. For the passengers the flight will be unpleasant.
C) The turbulence is a large scale one (waving) so that the aircraft will be
difficult to manoeuvre. The passengers will feel some discomfort.
D) The turbulence is wave like which makes the flight unpleasant for the
passengers but the manoeuvring will not be affected essentially.

1589. Turbulence and windshear may be encountered at low level up to ... km ahead of a mature
thunderstorm and up to ... feet.
A) 10 km ; 1000 ft
B) 32 km ; 6000 ft
C) 5 km ; 2000 ft
D) 14 km ; 3000 ft

1590. A zone of strong convection currents is encountered during a flight. In spite of moderate gust
you decide to continue the flight. What are your precautionary measures?
A) Increase the speed / try to descend below the zone of convective
currents.
B) Increase the speed / try to climb above the zone of convective currents, if
aircraft performance parameters allow.
C) Decrease the speed / try to climb above the zone of convective currents if
aircraft performance parameters allow.
D) Decrease the speed / try to descend below the zone of convective
currents.
1591. Clear Air Turbulence can be expected:
A) near a jet stream and around and above a CB cloud.
B) near a jet stream and in Stratus cloud.
C) with an upper level trough and in a col.
D) near a jet stream and with an anabatic wind.

1592. If crew and passengers are experiencing CAT which causes them to feel a definite strain against
their seat belts or shoulder straps, and food service and walking are difficult, with unsecured objects
being dislodged then it would be described as:
A) moderate.
B) light.
C) severe.
D) extreme.

1593.The degree of clear air turbulence experienced by an aircraft is proportional to the:


A) stability of the air.
B) intensity of the solar radiation.
C) intensity of vertical and horizontal windshear.
D) height of the aircraft.

1594. What hazard is likely to be encountered by a jet aircraft flying at 29.000ft, prior to overtaking
the surface position of a cold front?
A) Moderate to severe turbulence.
B) Cirrus.
C) Large Cumulus.
D) Rain ice.

1595. The diagram shown is a portion of a


significant weather chart in the Southern
Hemisphere. Flying at FL330 the worst
turbulence is likely to be encountered at:
A) C
B) B
C) A
D) D
1596. From the pre-flight briefing you know a jet stream is at 31,000 ft whilst you are at FL270. You
experience moderate CAT, what would be the best course of action?
A) Stay level.
B) Reduce speed.
C) Climb.
D) Descend.

1597. Turbulence is worst in a Jet stream:


A) along the axis of the core to the left.
B) in the core.
C) along the axis of the core to the right.
D) between the boundaries of the cold and warm air.

1598. What aviation hazards would you find in A2 or


A3?
A) CAT.
B) Rime ice.
C) Clear ice.
D) Turbulence in cloud.

1599. What is normally the most effective measure to reduce or avoid CAT effects?
A) Change of course.
B) Increase of speed.
C) Change of flight level.
D) Decrease of speed.
Thunderstorms:
1600. The expected inclusive time period of the initial stage of a thunderstorm is:
A) 1 hour.
B) 15 to 30 minutes.
C) 2 to 3 hours.
D) 1 ? hours.

1601. Where is a squall line to be expected?


A) In front of a cold front occlusion at higher levels.
B) At the surface position of a warm front.
C) Behind a cold front.
D) In front of an active cold front.

1602. Which of the options below would most likely lead to the formation of a thunderstorm?
A) An inversion, high absolute humidity, convective uplift.
B) Instability, convective uplift, high absolute humidifies.
C) Unstable air, convergent subsidence, a wide temperature and dewpoint
spread.
D) Very stable air, orographic uplift, increasing windspeed, saturated air.

1603. In Central Europe when is the greatest likelihood for thunderstorms due to warm updrafts?
A) Late morning.
B) Mid - afternoon.
C) Around midnight.
D) Early morning.

1604. During the life cycle of a thunderstorm, which stage is characterized predominantly by
downdrafts?
A) Dissipating stage.
B) Anvil stage.
C) Cumulus stage.
D) Mature stage.

1605. A microburst:
A) has a diameter up to 4 km.
B) has a life time of more than 30 minutes.
C) occurs only in tropical areas.
D) is always associated with thunderstorms.

1606. Thunderstorms can occur on a warm front if the:


A) cold air is moist and the environmental lapse rate is less than the dry adiabatic lapse rate.
B) cold air is moist and the environmental lapse rate exceeds the saturated adiabatic lapse rate.
C) warm air is moist and the environmental lapse rate exceeds the saturated adiabatic lapse rate.
D) warm air is moist and the environmental lapse rate is less than the saturated adiabatic lapse rate.
1607. A gust is a rapid increase in wind speed lasting ... and spread over ... A squall is a sudden
increase of wind speed of at least ... and lasting...
A) less than 1 minute, some distance, 16 kts, at least 1 minute.
B) less than 1 minute, a short distance, 16 kts, at least 1 minute.
C) at least 1 minute, some distance, 16 kts, less than 1 minute.
D) at least 1 minute, a short distance, 43 kts, less than 1 minute.

1608. Which of the following is considered to be one of the conditions necessary for thunderstorm
development?
A) ELR less than the DALR extending well above the 0 degree isotherm.
B) Instability throughout a deep layer extending well above the 0 degree isotherm.
C) A marked temperature inversion extending well above the 0 degree isotherm.
D) Stability throughout a deep layer extending well above the 0 degree isotherm.

1609. The most hazardous type of cloud that may be encountered on a cross country flight is:
A) cirrus.
B) cumulus.
C) cumulonimbus.
D) stratocumulus.
1610. Describe the microburst:
A) Precipitation at the base of a cumulonimbus cloud.
B) A high speed downburst of air with a generally lower temperature than its
surroundings.
C) Sinking air of higher temperature than its surroundings.
D) Horizontal movement of air, generally colder than its surroundings.

1611. Wind shear may be defined as:


A) a change in wind direction and/or speed, including up draughts and
downdraughts.
B) a sudden change in wind direction of 30degrees.
C) a vertical change in windspeed greater than 40 kt/1000ft.
D) a sudden increase of windspeed of at least 16 kt rising to 22 kt or more.

1612. A thunderstorm has the following stages in its life cycle:


A) cumulus of building, mature and dissipating.
B) building, mature, declining and dissipating.
C) cumulus, declining and dissipating.
D) cumulus, mature or building and declining.

1613. Which of the following weather factors will not cause the formation of thunderstorms?
A) An occlusion.
B) A warm front.
C) A cold front.
D) Sea fog.
1614. Which types of thunderstorm can be the most difficult to avoid?
A) Frontal thunderstorms.
B) Low-level thunderstorms.
C) Air mass thunderstorms.
D) Mature thunderstorms because they spread over such a wide area.
1615. What is a microburst?
A) Up- and downdraughts during a thunderstorm.
B) A downdraught, occurring when a warm front passes by.
C) Severe windshear, caused by a low level jet stream.
D) A downdraught of a CB in the mature stage.
1616. The fastest moving thunderstorms are:
A) Frontal.
B) Orographic.
C) Thermal.
D) Airmass.
1617. The formation of a cumulonimbus (CB) depends on:
A) Warm and humid air.
B) Some sort of lifting of the air.
C) all answers are correct.
D) Instability of thick layers of air.
1618. A microburst associated with a CB cloud will normally occur:
A) from a point mid-way up in the cloud.
B) only ahead the surface impact point.
C) near the base of the cloud.
D) anywhere up to 10 miles ahead of the cloud.
1619. Frontal thunderstorms are triggered off by:
A) mass ascent over large areas or cold air moving over a Warm surface.
B) rising air due to falling pressure or due to orographic uplift.
C) convection by intense daytime heating or cold air moving over a warm surface.
D) rising air due to falling pressure at air mass boundaries.
1620. There are two principal types of origin of thunderstorm.
A) Convection thunderstorms and frontal thunderstorms.
B) Convection thunderstorms and orographic uplift thunderstorms.
C) Air mass thunderstorms and frontal thunderstorms.
D) Airmass thunderstorms and heat thunderstorms.
1621. Strong windshear generating turbulence would be found:
A) to the left or right of a jet core.
B) always on the right of the jet core.
C) always on the left of the jet core.
D) directly beneath or above a jet core.
1622. The final stage of a thunderstorm is reached when:
A) a well developed anvil can be seen.
B) all of the above.
C) the lower portion of the cloud dissipates.
D) no further electrical charge is developed.
1623. Between the external appearance of a severe thunderstorm and the turbulence and hail that are
contained within it, there is:
A) Both b) and c) are correct.
B) A moderate correlation.
C) Apparently no correlation.
D) A strong correlation.
1624. In which of the following areas is the highest frequency of thunderstorms encountered?
A) Polar.
B) Tropical.
C) Temperate.
D) Subtropical.
1625. The most common hazards of thunderstorms to aircraft on or near the ground are:
A) turbulence and static.
B) lighting and hail.
C) heavy precipitation and surfaces squalls.
D) all of the above.
1626. What is a squall line?
A) Fog patches ahead of a warm front.
B) A band of cumulus clouds.
C) A band of intense thunderstorms.
D) Extended areas of nimbostratus clouds.
1627. Which thunderstorms are the more difficult to forecast and detect?
A) frontal thunderstorms.
B) cumulus thunderstorms.
C) air mass thunderstorms.
D) cumulus thunderstorms because they are smaller.
1628. Air mass thunderstorms are triggered off by:
A) fronts and/or orographic uplift.
B) convection at air mass boundaries.
C) standing waves in the lee of hills.
D) convection and/or orographic uplift.
1629. What is the size of a microburst?
A) Approximately 4 km in diameter.
B) A widespread area of approximately 10 km.
C) A small, clearly defined area of about 50 metres in diameter.
D) Approximately 200 metres in diameter.
1630. The Gust Front is:
A) formed by the cold outflow from beneath TS.
B) another name for a cold front.
C) directly below a TS.
D) characterised by frequent lightning.

1631. The building stage of a thunderstorm last for approximately:


A) 20 min
B) 20/30 min
C) 40 min
D) 30 min

1632. Thunderstorms reach their greatest intensity during the:


A) mature stage.
B) cumulus stage.
C) period in which precipitation is not falling.
D) dissipating stage.

1633. The gust front can be best described as being:


A) descending cold air from under a TS, undercutting warmer inflowing air.
B) descending warm air from under a CB, undercutting colder inflowing air.
C) rising warm air from under a TS, undercutting cold descending air.
D) rising cold air from under a TS, undercutting warmer descending air.

1634. During the life cycle of a thunderstorm, which stage is characterized predominately by
downdrafts?
A) Dissipating.
B) Cumulus.
C) Anvil.
D) Mature.

1635. Low level windshear is likely to be greatest:


A) at cloud base.
B) in fog.
C) below a surface inversion.
D) at the surface position of a cold front.

1636. The mature stage of a thunderstorm lasts for approximately:


A) 20/30 min
B) 40 min
C) 15 min
D) 30 min
1637. Microburst:
A) typically have horizontal dimensions 1 - 3km.
B) are always associated with CB clouds.
C) have an average lifespan of around 30mins.
D) only affect tropical areas.

1638. Isolated thunderstorms of a local nature are generally caused by:


A) frontal lifting (cold front).
B) thermal triggering.
C) frontal lifting (warm front).
D) frontal occlusion.

1639. The initial phase of a thunderstorm is characterized by:


A) continuous downdraughts.
B) continuous up draughts.
C) frequent lightning.
D) rain starting at surface.

1640. A super-cell thunderstorm requires the same start conditions as an ordinary thunderstorm, and
in addition:
A) a warm front nearby.
B) a cold front nearby.
C) a lot of moisture and a wind vector change aloft.
D) cold, dry air at low level.

1641. What is a microburst?


A) A small low pressure system where the wind circulates with very high
speeds.
B) A concentrated downdraft with high speeds and a lower temperature than
the surrounding air.
C) An extremely strong wind gust in a tropical revolving storm.
D) A concentrated downdraft with high speeds and a higher temperature
than the surrounding air.

1642. The most likely trigger for an air mass type of thunderstorm is:
A) intense insolation of a land surface in the area of a col or weak low.
B) secondary low forming on a trailing cold front.
C) advection of warm air over a relatively cold sea or land surface.
D) lifting at a cold front at night.

1643. Which of the following statements is true with regard to a mature thunderstorm:
A) the cloud top assumes and anvil shape.
B) water droplets, hail and snow are all present.
C) the top of the cloud is negatively charged and the base is positively charged.
D) both up-currents and down-currents appear concurrently.
1644. Which statement is true with regard to a thunderstorm?
A) Hail must be expected in all stages.
B) Up- and downdrafts can occur in CU and CB clouds.
C) Only downdrafts may be expected in the mature and dissipating stage
(CB) of a thunderstorm.
D) Updrafts in CU clouds, up- and downdrafts as well as downdrafts only in
the CB (mature and dissipating stage).

1645. Isolated TS occur mostly due to:


A) Warm frontal uplift.
B) Cold front uplift.
C) Insolation.
D) Convection.

1646. A microburst phenomenon can arise in the:


A) updraught of a cumulonimbus at the mature stage.
B) updraught of a cumulonimbus at the growth stage.
C) downdraught of a cumulonimbus at the formation stage.
D) downdraught of a cumulonimbus at the mature stage

1647. What feature is normally associated with the cumulus stage of a thunderstorm?
A) Rain or hail at the surface.
B) Frequent lightning.
C) Roll cloud.
D) Continuous updraft.

1648. Which of the following statements is correct about thunderstorms?


A) They usually move in relation to the 10,000ft/700mb wind.
B) At the cumulus stage mainly downdraughts are in evidence.
C) Windshear and hail occur only in the cumulus stage.
D) They are often associated with lenticularis cloud.

1649. In a thunderstorm pilots should be on alert for hail when radar echoes:
A) None of the above is correct.
B) are well defined.
C) are ill-defined.
D) show turbulence.

1650. What is the location of a squall line?


A) Ahead of a warm front.
B) Between cold and warm front.
C) Ahead of a cold front.
D) On the rear side of a frontal system.
1651. The diameter and the life time of a typical microburst are in the order of:
A) 12 km and 5-10 minutes.
B) 8 km and 5-15 minutes.
C) 4 km and 1-5 minutes.
D) 4 km and 30-40 minutes.

1652. In which stage of the life cycle of a single thunderstorm cell occur both up- and downdrafts
simultaneously?
A) Mature stage.
B) Cumulus stage.
C) Dissipating stage.
D) In all stages.

1653. Which weather phenomenon signals the beginning of the mature stage of a thunderstorm:
A) Growth rate of cloud is maximum.
B) The start of strong wind gusts.
C) The start of precipitation.
D) The appearance of an anvil top.

1654. When are thunderstorms most likely in Europe?


A) Midday.
B) Just after dawn.
C) Midnight.
D) Late afternoon.

1655. In addition to a lifting action, what are two other conditions necessary for thunderstorm
formation?
A) Unstable conditions and low atmospheric pressure.
B) Stable conditions and high moisture content.
C) Stable conditions and low atmospheric pressure.
D) Unstable conditions and high moisture content.

1656. The start of the mature stage of a thunderstorm is identifiable from:


A) very strong up-draughts and a cauliflower shaped cloud top.
B) lightning.
C) extensive light down-draughts, precipitation and lightning.
D) the onset of precipitation and the " fibrous" top to the cloud.

1657. In the building stage of a thunderstorm, which of the following is true?


A) Only up-currents are present.
B) Up-currents and side-currents are present.
C) Only down-currents are present.
D) Both up-currents and down-currents are present.
1658. When would a rotor cloud be ahead of a Cb?
A) Initial stage.
B) Mature stage.
C) Dissipating stage.
D) Cumulus stage.

1659.What are the meteorological prerequisites, at low level, for thunderstorms formed by lifting
processes, over land?
A) Subsidence, inversion.
B) High air pressure (> 1013 hPa), high temperatures.
C) Low temperatures, low humidity.
D) High temperatures, high humidity.

1660. What is a microburst?


A) Air descending at high speed, the air is colder than the surrounding air.
B) A small depression with high wind speeds.
C) A small Tropical Revolving Storm.
D) Air is descending at high speed, the air is warmer than the surrounding
air.

1661. Up and down going draughts in a thunderstorm occur in which stage?


A) Cumulus stage.
B) Dissipating stage.
C) Mature stage.
D) Precipitation stage.

1662. At which altitude, at temperate latitudes, may hail be expected in connection with a CB?
A) From the ground up to about FL 200.
B) From the ground up to about FL 100.
C) From the base of the clouds up to FL 200.
D) From the ground up to a maximum of FL 450.

1663. What is true about thunderstorms?


A) They only occur, when a cold front occlusion is present.
B) They can exceptionally occur at a warm front, when warm air is unstable.
C) They occur only in the summertime.
D) They occur at cold fronts only.

1664. During their life single-cell thunderstorms move according to the ... winds
A) 200mb
B) 500mb
C) 700mb
D) 1000mb
1665. The alternative name for airmass thunderstorms is:
A) convective thunderstorms.
B) heat thunderstorms.
C) CB thunderstorms.
D) detached thunderstorms.

1666. Aircraft struck by lightning may sometimes get considerable damage and at least temporarily
the manoeuvring of the aircraft will be made more difficult. Which one of the following statements is
correct?
A) Aircraft made by composite material can’ t conduct a lightning and will
therefore very seldom be struck.
B) Aircraft made by composite material may get severe damage, the crew
may be blinded and temporarily lose the hearing.
C) An aircraft made by metal has a certain capacity to attract a lightning,
but the lightning will follow the surface and therefore no damage will be
caused.
D) An aircraft has in the atmosphere the same qualities as a " Faradays
cage" , which means that struck of lightning seldom occurs. But if it
happens, the result will be an occasional engine failure. The crew may get
a shock.

1667. The expected inclusive time period from the development to dissipation stage of a thunderstorm
is:
A) two to three hours.
B) one hour 30 mins.
C) half an hour.
D) one hour.

1668. The hazards that thunderstorms pose to aviation include:


A) Windshear, severe turbulence, hail.
B) All of the above.
C) Severe icing.
D) Heavy precipitation, lightning.

1669. Where can wind shear associated with a thunderstorm be found? (Choose the most complete
answer).
A) On all sides of the thunderstorm cell and directly under the cell.
B) In front of the thunderstorm cell and directly under the cell.
C) Ahead of the roll clouds or gust front.
D) In front of the thunderstorm cell (anvil side).

1670. What are the requirements for the formation of a thunderstorm?


A) Sufficient moisture, an unstable lapse rate, and a lifting action.
B) Sufficient water vapor and a lifting action.
C) A cumulus cloud with sufficient moisture.
D) A cumulus cloud with sufficient moisture and an inverted lapse rate.
1671. What are squall lines?
A) The surface weather associated with upper air troughs.
B) The paths of tropical revolving storms.
C) Unusual intensive cold fronts.
D) Bands of intensive thunderstorms.

1672. Which thunderstorms generally produce the most severe conditions, such as heavy hail and
destructive winds?
A) Warm front.
B) Squall line.
C) Cold front.
D) Air mass.

1673. In general terms, an intense contour low will indicate:


A) heavy rain, hail and thunderstorm.
B) no change to the weather.
C) quiet settled weather.
D) changeable weather but getting better.

1674. Continuous up draughts occur in a thunderstorm during the:


A) period in which precipitation is falling.
B) dissipating stage.
C) cumulus stage.
D) mature stage.

1675. Which weather phenomenon is always associated with a thunderstorm?


A) Lightning.
B) Heavy rain showers.
C) Hail.
D) Supercooled raindrops.

1676. Which of the following conditions are necessary for thunderstorms?


A) High moisture content and a trigger action.
B) Low surface temperatures and high moisture content.
C) Instability at night.
D) Low upper temperatures and a high 0 degree isotherm.

1677. During which stage of thunderstorm development are rotor winds characterized by roll clouds
most likely to occur?
A) Mature stage.
B) Cumulus stage and mature stage.
C) Cumulus stage.
D) Dissipating stage.
1678. Which of the following statements describes a microburst?
A) An extremely strong wind gust associated with a tropical revolving storm.
B) A high speed downdraft of air with a higher temperature than its
surroundings.
C) A small low pressure system where the wind circulates at high speed.
D) A high speed downburst of air with a generally lower temperature than its
surroundings.

1679. Air mass thunderstorms:


A) have a marked seasonal variation in equatorial latitudes.
B) have a marked diurnal variation.
C) can give intermittent rain.
D) tend to develop along a well marked line.

1680. The expected inclusive time period from development to dissipation of a thunderstorm is:
A) 30 minutes.
B) 1 hour.
C) 2 to 3 hours.
D) 24 hours.

1681. What are the requirements for the formation of a thunderstorm?


A) Water vapour and high pressure.
B) A cumulus cloud with sufficient moisture associated with an inversion.
C) A stratocumulus cloud with sufficient moisture.
D) An adequate supply of moisture, conditional instability and a lifting
action.

1682. What type of thunderstorms develop in the afternoon in summer over land in moderate
latitudes?
A) Cold mass thunderstorms.
B) Airmass thunderstorms.
C) Warm front thunderstorms.
D) Occlusion thunderstorms.

1683. Which thunderstorms generally develop in the afternoon in summer over land in moderate
latitudes?
A) Airmass thunderstorms.
B) Occlusion thunderstorms.
C) Cold mass thunderstorms.
D) Warm front thunderstorms.

1684. A gustfront is:


A) formed by the cold air outflow from a thunderstorm.
B) characterized by heavy lightning.
C) normally encountered directly below a thunderstorm.
D) another name for a cold front.
1685. Which of the following are not essential factors for thunderstorm development?
A) All above mentioned are essential.
B) A supply of moist air.
C) An unstable atmosphere.
D) High temperature.

1686. Where does the largest chance of squalls occur?


A) Above the occlusion along the cold front.
B) Behind the cold front.
C) Above the occlusion along the warm front.
D) In front of an active cold front.

1687. What is the approximate maximum diameter of a microburst?


A) 4 km.
B) 20 km.
C) 400 m.
D) 50 km.

1688. In temperature latitudes frontal thunderstorms:


A) have a large diurnal variation in summer.
B) have a marked seasonal variation.
C) have a large diurnal variation in winter.
D) persist longer than airmass thunderstorms.
1689. If you cannot avoid penetrating a thunderstorm, which is the best area to penetrate?
A) The bottom.
B) The top.
C) The sides.
D) The middle.

1690. How long does a typical microburst last?


A) 1 to 2 hours.
B) About 30 minutes.
C) Less than 1 minute.
D) 1 to 5 minutes.

1691. Which of the following meteorological phenomenon indicates upper level instability which may
lead to thunderstorm development?
A) Red cirrus.
B) Halo.
C) AC castellanus.
D) AC lenticularis.

1692. What weather condition would you expect at a squall line?


A) Thunderstorms.
B) Fog.
C) Strong whirlwinds reaching up to higher levels.
D) Strong steady rain.

1693. Concerning the radar reflectivity in relation to a thunderstorm, the following is true:
A) Reflectivity is a function of the number and size of water droplets in a
given unit of volume.
B) Reflectivity decreases with severity and frequency of turbulence.
C) Reflectivity increases with severity and frequency of turbulence.
D) Both b) and c) are correct.

1694. Which thunderstorms move forward the fastest?


A) Thermal thunderstorms.
B) Orographic thunderstorms.
C) Thunderstorms formed by lifting processes.
D) Frontal thunderstorms.

1695. The average wind vector change - headwind to tailwind - encountered in microbursts is ... and
the maximum recorded is...
A) 100kt; 200kt.
B) 50kt; 100kt.
C) 25kt; 40kt.
D) 25kt; 75kt.
Tornadoes:
1696. Which of the following statements apply to tropical revolving storms which affect the coast of
Florida:
A) called typhoons and occur from June to November.
B) called hurricanes and occur from August to October.
C) called hurricanes and occur from January to May.
D) called cyclones and occur from January to March.

1697. In a tornado the maximum core diameter can reach ... and wind speeds of ... have been
recorded.
A) 100m; over 300mph
B) a few hundred metres; over 200kt
C) 50m; over 200kt
D) 1000m; over 300kt

1698. At what time of the year are tornadoes most likely to occur in North America?
A) Autumn, winter.
B) Summer, autumn.
C) Spring, summer.
D) Winter.

1699. Tornadoes in the USA occur mainly in ... with a peak occurrence at about...
A) autumn; 1500.
B) spring; 1700.
C) winter 0000.
D) august; 1500.

1700. A typical tornado diameter is:


A) 2 - 6 km.
B) more than 10kms.
C) less than 100m.
D) 100 - 150m.

1701. The diameter of a typical tornado is:


A) 100 to 150 metres.
B) in the order of 10 km.
C) only a few metres.
D) about 2 to 6 km.

1702. With which type of cloud are tornadoes normally associated?


A) Cumulonimbus.
B) Stratus.
C) Nimbostratus.
D) Cumulus.
Low and high level inversion:
1703. Climbing out of Dhahran, Saudi Arabia on a clear night you suddenly lose your rate of climb.
Why?
A) Engine full of sand.
B) Marked temperature inversion.
C) Downdraft.
D) VSI blocked.

Hazards in mountainous area:


1704. Which conditions lead to mountain waves?
A) Stable air, speed , > 20 kts across the ridge.
B) Unstable moist air, speeds < 5 kts across the ridge.
C) Stable air, speed > 30kts, parallel to the ridge.
D) Unstable air, speed > 20 kts across the ridge.

1705. When flying in IMC close to a range of hills 2,000 ft high, in stable air and with wind direction at
right angles to the axis of the range of hills, which of the following is probably the most dangerous
practice?
A) Flying parallel to the hills on the upwind side at FL55.
B) Flying towards the hills, into the wind, at FL65.
C) Flying towards the hills downwind at FL55.
D) Flying parallel to the hills on the downwind side at FL25.

1706. With reference to rotor streaming:


A) it is another name for the rotor zones associated with mountain waves.
B) it can occur at levels up to 80.000 ft.
C) it is found at approximately the tropopause level.
D) it is a low level phenomenon.

1707. When would you encounter hoar frost?


A) Ns.
B) Descending into cold air.
C) Climbing through an inversion.
D) Cb.

1708. The worst turbulence associated with mountain waves will be experienced:
A) in lenticularis cloud at the tropopause.
B) in lenticularis cloud above the rotor cloud.
C) in the rotor zone.
D) in the cap cloud.
1709. Conditions favourable for the formation of mountain (lee) waves are:
A) approximately a 90° change in wind direction with height, wind blowing
within 30° of the perpendicular to the ridge, wind speed at the crest
above 15 kts for small mountains and above 30 kts for large mountains..
B) little change in wind direction with height, wind speeds increasing
upwards through the troposphere, wind blowing within 30° of the
perpendicular to the ridge, wind speed at the crest above 15 kts for small
mountains and above 30 kts for large mounta
C) approximately a 90° change in wind direction with height, wind speeds
increasing upwards through the troposphere, wind blowing within 30° of
the perpendicular to the ridge, wind speed at the crest above 15 kts for
small mountains and above 30 kt
D) little change in wind direction with height, wind speeds increasing
upwards through the troposphere, wind blowing within 090° of the
perpendicular to the ridge, wind speed at the crest above 15 kts for small
mountains and above 30 kts for large mount

1710. The meteorological conditions required to form mountain waves are among others:
A) The height of the mountain must be at least 3000 ft.
B) The air must be unstable.
C) Wind velocity on top of the mountain must be 50 kts or more.
D) A wind profile whereby the wind velocity increases with increasing
altitude and a strong, steady wind flow at higher levels often extending to
the tropopause.

1711. A north/south mountain range, height 10.000 ft, is producing marked mountain waves. For the
formation of powerful mountain waves, the air at the level of the ridge and the first few thousand feet
above the ridge was:
A) conditionally stable.
B) unstable.
C) stable with less stable layers above and below.
D) unstable with stable air above and below.

1712. What is likely when mountain waves are observed?


A) Convection clouds.
B) Strong wind and turbulence in the mountains.
C) Unstable air.
D) Rain on the leeward side of the mountain.

1713. You intend to carry out a VFR flight over the Alps, on a hot summer day, when the weather is
unstable. What is the best time of day to conduct this flight?
A) Morning.
B) Early evening.
C) Afternoon.
D) Mid-day.
1714.A north/south mountain range, height 10.000 ft, is producing marked mountain waves. The
greatest potential danger exists for an aircraft flying:
A) above a line of clouds parallel to the ridge at FL 250.
B) at FL 350 over and parallel to the ridge.
C) on the windward side of the ridge.
D) towards the ridge from the lee side at FL 140.

1715. CAP clouds are one three groups of mountain wave clouds.
A) Both a) and b) are correct.
B) They appear to be harmless.
C) Downdrafts as great as 5000 feet per minute can be expected on the
leeward side.
D) They cool adiabatically and therefore evaporate.

1716. For the formation of mountain waves, the wind above the level of the ridge should:
A) increase and then reverse in direction.
B) decrease or even reverse direction.
C) increase initially, then decrease.
D) increase with little change in direction.

1717. Should a mountain wave situation occur without creating mountain wave clouds, this would
indicate:
A) Stable air mass.
B) Too strong winds.
C) Too dry air.
D) Too strong turbulence.

1718. Which of the following conditions are most favourable to the formation of mountain waves?
A) Stable air at mountain top altitude and a wind at least 20 knots blowing
across the mountain ridge.
B) Moist unstable air at mountain top and wind of less than 5 knots blowing
across the mountain ridge.
C) Either stable or unstable air at mountain top and a wind of at least 30
knots blowing parallel to the mountain ridge.
D) Unstable air at mountain top altitude and a wind at least 20 knots
blowing across the mountain ridge.

1719. In relation to mountain wave activity, which of the following cloud types signifies the most
turbulent conditions?
A) Roll or rotor clouds.
B) Cumulus clouds around the mountain tops.
C) Lenticularis clouds.
D) Stratiform clouds, covering the mountain tops.
1720. You are flying with an outside air temperature of -12° C and a TAS of 250 kt at FL 150 through
8 octas NS. What type and degree of icing is most probable?
A) Over flat terrain, moderate hoar frost.
B) Over flat terrain, away from fronts, moderate to severe mixed ice.
C) In clouds pushed up against the mountains, moderate to severe mixed
ice.
D) In clouds pushed up against the mountains, moderate to severe rime ice.

1721. In a mountain wave environment:


A) downwind flight may not be safe, as height variations are out of phase
with the waves.
B) when flying upwind, the aircraft is liable to be at its maximum height,
when over high ground.
C) upwind flight may not be safe, as height variations are usually out of
phase with the waves.
D) downwind flight is safer as height variations are out of phase with the
waves.

1722. Which of the following phenomena are formed when a moist, stable layer of air is forced to rise
against a mountain range?
A) Inversions.
B) Showers and thunderstorms.
C) Stratified clouds.
D) Areas of severe turbulence.

1723. Mention 3 types of weather phenomena associated with mountain waves:


A) Fog, lenticularis and medium height clouds.
B) Rotor , cap and lenticularis clouds.
C) Low clouds, downdrafts and thunderstorms.
D) Turbulence, advection and convection clouds.
Visibility reducing phenomena:
1724. In general, the meteorological visibility during rainfall compared to during drizzle is:
A) less.
B) the same.
C) greater.
D) in rain - below 1 km, in drizzle - more than 2 km.

1725. The forms of visible water which reduce visibility in the atmosphere are:
A) cloud, fog, mist, spray or smog.
B) fog, spray, cloud and hail (but only for short periods).
C) cloud, fog, smog, spray or precipitation.
D) cloud, fog, mist spray or precipitation.

1726. Compare meteorological visibility:


A) Visibility is greater in FG than in DZ.
B) Visibility is equal in RA and in DZ.
C) Visibility is lower in RA than in DZ.
D) Visibility is greater in RA than in DZ.

1727. R23L/P1500. What does this imply?


A) RVR is persistently 1500m.
B) RVR is more than 1500m.
C) RVR is improving.
D) RVR is unchanged during the last 10 mins.

1728. In unstable air, surface visibility is most likely to be restricted by:


A) showers of rain or snow.
B) low stratus.
C) haze.
D) drizzle.

1729. Reduction in visibility due to precipitation depends on:


A) precipitation type and duration.
B) precipitation intensity and type.
C) precipitation intensity and duration.
D) precipitation duration and droplet size.

1730. R23L/P1200. What does this imply?


A) RVR is unchanged during the last 10 minutes.
B) RVR is more than 1200m.
C) RVR is persistently 1200m.
D) RVR is improving.
1731. In the vicinity of industrial areas, smoke is most likely to affect surface visibility when:
A) there is a low level inversion.
B) the surface wind is strong and gusty.
C) a rapid moving cold front has just passed the area.
D) cumulus clouds have developed in the afternoon.

1732. Flight visibility from the cockpit during approach in a tropical downpour can decrease to
minimal:
A) about 500 metres.
B) about 1000 metres.
C) about 200 metres.
D) tens of metres.

1733. The flight visibility from the cockpit of an aircraft in a tropical downpour can decrease to
minimal...
A) about 1000 metres.
B) about 500 metres.
C) tens of metres.
D) about 200 metres.

1734. The solid particles which reduce visibility in the atmosphere are:
A) atmospheric pollution, volcanic fumes, dust and sand.
B) atmospheric pollution, dust, sand and volcanic ash.
C) atmospheric pollution, dust, ozone and moisture.
D) atmospheric pollution, dust, sand and spray.

1735. Visibility is reduced by haze when:


A) small water droplets are present.
B) a light drizzle falls.
C) dust particles are trapped below an inversion.
D) a cold front just passed.
Meteorological information
Observation:
1738. In which direction
does the polar front move
in this picture?
A) D
B) B
C) C
D) A

1739. If a cold front moves at 30 knots, you could expect the warm front to be moving at:
A) 40 knots.
B) 30 knots.
C) 50 knots.
D) 20 knots.

1740. Where does the frontal depression move in the direction of the 2000 ft wind?
A) Ahead of the warm front.
B) In the warm sector.
C) In the center of an occlusion.
D) At the rear side.

1742. On a summer day, the following observations are made in Zurich:


0450Z 24009 KT 7000 SCT040 SCT120 15/12 Q1014 NOSIG =
0650Z 24010KT 6000 SCT040 SCT120 17/13 Q1012 NOSIG =
0850Z 23014KT 8000 BKN100 19/13 Q1009 BECMG 26020G35KT TS =
1050Z 28022G33KT 4000 TSRA SCT015 SCT050CB OVC080 16/14 Q1006 BECMG NSW = 1250Z
31016KT 9999 SCT025TCU BKN030 13/09 Q1009 NOSIG =
1450Z 30012KT 9999 SHRA BKN020TCU 14/10 Q1011 NOSIG =
1650Z 30009KT SCT025 BKN035 13/10 Q1013 RESHRA NOSIG =
1850Z 28006KT 9999 SCT040 11/09 Q1014 NOSIG =
2050Z 26004KT CAVOK 10/08 Q1015 NOSIG =
You conclude, that...
A) a warm front has passed the station in the morning, and warm sector
weather prevailed in the afternoon.
B) a cold front has passed the station in the morning, and rear side weather
prevailed in the afternoon.
C) a cold front in the morning, and a warm front in the afternoon have
passed the station.
D) a warm front in the morning, and a cold front in the afternoon have
passed the station.
1743. On a summer day, the following observations are made in Zurich:
0450Z 23015KT 3000 RA SCT008 SCT020 0VC030 13/12 Q1010 NOSIG= 0650Z 25008KT 6000
SCT040 BKN090 18/14 Q1010 RERA NOSIG=
0850Z 25006KT 8000 SCT040 SCT100 19/15 Q1009 NOSIG=
1050Z 24008KT 9999 SCT040 SCT100 21/15 Q1008 NOSIG=
1250Z 23012KT CAVOK 23/16 Q1005 NOSIG=
1450Z 23016KT 9999 SCT040 BKN090 24/17 Q1003 BECMG 25020G40KT TS= 1650Z 24018G35KT
3000 TSRA SCT006 BKN015CB 18/16 Q1002 NOSIG= 1850Z 28012KT 9999 SCT030 SCT100 13/11
Q1005 NOSIG=
You conclude, that...
A) a trough line early in the morning and a warm front late in the afternoon
have passed the station.
B) a warm front early in the morning and a cold front late in the afternoon
have passed the station.
C) a cold front early in the morning and a warm front late in the afternoon
have passed the station.
D) an air mass thunderstorm only has passed during the entire day.

1744. In which direction does


the polar front move in this
picture?
A) C
B) B
C) A
D) D

1746. What is a SPECI?


A) A routine aerodrome weather report issued every 3 hours.
B) A warning of meteorological dangers at an aerodrome, issued only when required.
C) An aerodrome forecast issued every 9 hours.
D) A selected special aerodrome weather report, issued when a significant
change of the weather conditions have been observed.

1747. Wind from Air Traffic Control is 035° /30 kts. If the QDM of the runway is 083° and the variation
is 20° E the headwind component will be:
A) 23 kts.
B) 32 kts.
C) 26 kts.
D) 20 kts.

1748. Satellite images are used to:


A) locate wind currents on the ground.
B) locate precipitation zones.
C) to achieve 14 day forecasts.
D) locate fronts in areas with few ground stations.
1749.
Airfield X is 69Mtrs below sea level, QFF is 1030Hpa, temperature is ISA -10 deg C. What is the QNH?
A) 1030Hpa.
B) More than 1030Hpa.
C) Less than 1030Hpa.
D) Impossible to tell.

1750. The greatest meteorological hazard for landing at VGZR is likely to be:

A) gusting surface wind,


B) low cloud.
C) wind shear.
D) poor visibility.

1751. Clouds are grouped into ... basic cloud genera and cloud cover is measured in...
A) 10; oktas.
B) 10; tenths.
C) 8; oktas.
D) 4; oktas.

1752. Area forecasts for low-level flights exchanged between meteorological offices in support of
issuance of AIRMET information are prepared in standard format. When plain language is used, the
forecast is known as a GAMET area forecast. The horizontal visibility reported in such a forecast refers
to:
A) the flight visibility below clouds.
B) surface visibility.
C) the meteorological visibility up to 500 m.
D) the average flight visibility of the distance in question.

1753. What are the images of satellites provided daily by the Weather Service used for?
A) To help provide 14-day forecasts.
B) To measure wind currents on the ground.
C) To locate precipitation zones.
D) To locate fronts in areas with few observation stations.
1754. Aerodrome QNH is the:
A) lowest value of QNH forecast to occur in the region of the aerodrome
during a period of one hour.
B) aerodrome pressure converted to mean sea level assuming ISA
conditions between the aerodrome and sea level.
C) atmospheric pressure at the aerodrome level.
D) QFE converted to mean sea level assuming constant temperature
conditions between the aerodrome and sea level.

1755. A cup ... and remote transmitting vane form the transmitting head of the electrical ... which
enables a continuous record of wind direction and speed to be made on a moving chart.
A) anemometer; barograph
B) anemograph; anemometer
C) anemometer; anemograph
D) barograph; anemometer

1756. LFCG 221100Z 1219 22010KT 4500 RA BKN010 OVC015 TEMPO 1500 + RA OVC005 FM1430
29020G35KT 6000 SHRA BKN008TCU OVC015 TEMPO 1619 25010 KT 9999 NSW BKN020 PROB30
TEMPO 1619 1500 TSGR BKN007CB
What is the lowest cloud base you may encounter at 1345Z?
A) 700 ft
B) 1000 ft
C) 500 ft
D) 12.000 ft

1757. The wind indicator for a weather observation receives the measured value from an anemometer.
Where is this instrument placed?
A) Close to the station about 2 m above the ground.
B) 1 m above the runway.
C) On a mast 8-10 m above the ground.
D) On the roof of the weather station.

1758. EGCC 150820Z VRB03KT 3500N FU SKC 08/07 Q1024 TEMPO 0800 BCFG
For most of the validity period of this message, the poorest visibility will be found:
A) Impossible to tell.
B) In the BCFS.
C) To the north of the airport.
D) To the south of the airport.

1759. On the Boeing 737-400 EFIS EHSI, on which modes is a Weather Radar Display available?
A) MAP, EXP VOR/ILS, EXP NAV.
B) MAP, FULL VOR/ILS, FULL NAV.
C) MAP, PLAN, EXP VOR/ILS.
D) MAP, PLAN, FULL VOR/ILS.
1760. LFCG 221100Z 1219 22010KT 4500 RA BKN010 OVC015 TEMPO 1500 + RA OVC005 FM1430
29020G35KT 6000 SHRA BKN008TCU OVC015 TEMPO 1619 25010 KT 9999 NSW BKN020 PROB30
TEMPO 1619 1500 TSGR BKN007CB
What is the most likely visibility for landing at 1345Z?
A) 6.000 m
B) 1.500 m
C) 12.000 m
D) 4.500 m

1761. The surface wind velocity at DELHI (VIDP) at 0200 UTC is:

A) 240 (T) 8 knots gusting to 18 knots.


B) calm.
C) 240 (M) 8 knots gusting 18 knots.
D) light and variable.

1762. The three elements in a cloud report are:


A) amount, cover, height of base.
B) amount, type, height of base.
C) type, cover, amount.
D) type, amount, cloud species.

1763. Precipitation and turbulence areas are coded by colour, amber indicates:
A) 12 – 50 mm/hr
B) 1 – 4 mm/hr
C) 4 – 12 mm/hr
D) 0 - 1 mm/hr

1765. When is the RVR reported at most airports?


A) When the meteorological visibility decreases below 800 m.
B) When the RVR decreases below 800 m.
C) When the meteorological visibility decreases below 1500 m.
D) When the RVR decreases below 2000 m.
1766. The meaning of RVR is:
A) cross-wind component.
B) runway visual range.
C) braking action.
D) meteorological visibility.
1767. Normally, temperature readings are taken at a height of ... in a Stevenson Screen:
A) 10 m
B) 40 cm
C) 4 ft (1.25 m)
D) 4 m
1768. EGCC 150820Z VRB03KT 3500N FU SKC 08/07 Q1024 TEMPO 0800 BCFG
The w/v reported at this airport is:
A) Speed and direction variable, maximum speed recorded as 3 knots.
B) Variable direction, speed 3 knots.
C) Variable wind direction averaging 3 knots over the last 10 minutes.
D) Calm.
1769. Given : True altitude 9.000 FT, OAT -32° C, CAS 200 kt
What is the TAS?
A) 215 kt
B) 220 kt
C) 200 kt
D) 210 kt
1770. Surface air temperature is recorded at a height of:
A) 4 feet
B) 1 inch
C) 1 metre
D) 1 foot

1771. Using the radiosonde diagrams, which


would most likely show ground fog?
A) 4
B) 2
C) 3
D) 1
1772. When last 10 minutes wind velocity deviates more than 10 kts from mean value during the
same period of time, this will be indicated as:
A) tempo 10 kts
B) geostrophic wind.
C) a gust
D) a squall

1773. Occasionally a frost thermometer will be placed ... at a point ... above short cut grass:
A) vertically 4 feet.
B) vertically 40 mm.
C) horizontally 40 cm.
D) horizontally 40 mm.

1774. EGCC 150820Z VRB03KT 3500N FU SKC 08/07 Q1024 TEMPO 0800 BCFG
The temperatures and pressure at the airport are:
A) Dry bulb +8° C, dew point +7° C, QFE 1024mb.
B) Dry bulb +8° C, wet bulb +7° C, QFE 1024mb.
C) Dry bulb +8° C, dew point +7° C, QNH 1024mb.
D) Dry bulb +8° C, wet bulb +7° C, QNH 1024mb.

1775. EGCC 150820Z VRB03KT 3500N FU SKC 08/07 Q1024 TEMPO 0800 BCFG
For a landing at 0845Z, what are the worst conditions you may encounter?
A) Sky covered.
B) 3.500 m in haze.
C) 8 km in shallow fog.
D) 800 m in fog patches

1776. An airborne weather radar installation makes it possible to detect the location of:
A) zones of precipitation, particularly liquid-state precipitation, and also their
intensity.
B) stratocumulus and its vertical development.
C) cumulonimbus, but provided that cloud of this type is accompanied by
falls of hail.
D) all clouds.

1777. Under what conditions is RVR measured?


A) When the meteorological visibility is lower than 1500 metres.
B) When fog is present.
C) When the technical equipment for the measurement at an airport is
available.
D) When the meteorological visibility is lower than 1000 metres.
1778. The forecast visibility at ALEXANDRIA 9HEAX0 at 1000 UTC is:

A) 10 km or more.
B) changing steadily from 8.000 m to 4.000m.
C) 4.000 m.
D) 8.000 m.

1779. EGCC 150820Z VRB03KT 3500N FU SKC 08/07 Q1024 TEMPO 0800 BCFG
For how long is this weather message valid?
A) 1 hour
B) 2 hours
C) 30 minutes
D) 24 hours

1780. The forecast lowest cloud at DACCA (VGZR) at 2300 UTC is

A) 2-4 oktas at 300 ft aal.


B) 2-4 oktas at 300 ft amsl.
C) 5-7 oktas at 300 ft aal.
D) 5-7 oktas at 300 ft amsl.
1781.EGCC 150820Z VRB03KT 3500N FU SKC 08/07 Q1024 TEMPO 0800 BCFG
What is causing a visibility of 3.500 m?
A) Smoke
B) Haze
C) Mist
D) Fog

1782. What is a SPECI?


A) A landing forecast.
B) A report produced when significant changes have occurred.
C) A forecast and valid for 6 hours.
D) A forecast valid for 3 hours.

1783. Which of the following causes echoes on meteorological radar screens?


A) Any cloud.
B) Fog.
C) Hail.
D) Water vapour.

1784. At an airport various types of visibility values are reported. One of these is determined by the
observer by means of marks and/or lights at known distances. What is this known as?
A) meteorological visibility.
B) slant range visibility.
C) runway visual range.
D) vertical visibility.

1786. What does a Transmissiometer measure?


A) Breaking action.
B) Cloud base.
C) Wind speed.
D) Visibility.

1787. Reading of the wet and dry bulb thermometers can be used to obtain:
A) saturation vapour pressure, dew point and relative humidity.
B) absolute humidity, general humidity and relative humidity.
C) relative humidity, dew point and absolute humidity only.
D) relative humidity, absolute humidity, dew point and air temperature.

1788. An instrument for measuring humidity is known as:


A) a pair of wet and dry bulb thermometers.
B) all of the above.
C) a hygrometer.
D) a psychrometer.
1789. Which of the following meteorological phenomena can rapidly change the braking action of a
runway?
A) HZ
B) FG
C) +FZRA
D) MIFG

1790. The four temperatures normally recorded in a Stevenson Screen are as follows:
A) air temperature, dry bulb, wet bulb, maximum/minimum.
B) dry bulb, wet bulb, maximum, minimum.
C) dry bulb, wet bulb, maximum/minimum and frost.
D) air temperature, dry bulb, wet bulb, and hygrometer

1791. RVR is:


A) measured using ceilometers along the runway.
B) displayed in TAFs and METARs.
C) usually greater than met visibility.
D) given when the met visibility is below 2000m.

1792. ACARS is:


A) a means of automatically transmitting weather information from aircraft to meteorological stations.
B) automated VOLMET messages from ground to air.
C) a means of automatically transmitting weather information from meteorological stations to aircraft.
D) a means of transmitting operational messages including TAFs/METARs from ground to air.

1793. If you fly at right angles to a jet stream in Europe with a decreasing outside air temperature,
you will experience...
A) port drift.
B) starboard drift.
C) increasing groundspeed.
D) decreasing groundspeed

1794. What does RVR stand for?


A) Radio VHF and range.
B) Runway visual range.
C) Reduced vertical range.
D) Reduced visibility range.

1795. While approaching your target aerodrome you receive the following message: RVR runway 23:
400m This information indicates the:
A) portion of runway which a pilot on the threshold of any of the runways would
see, with runway 23 in service.
B) meteorological visibility on runway 23.
C) length of runway which a pilot in an aircraft on the ground would see, on the
threshold of runway 23.
D) minimum visibility at this aerodrome, with runway 23 being the one in service.
1796. Barometric tendency is measured on ... and recorded on...
A) barograph; aneroid barometer.
B) an anemometer; an aneroid barometer.
C) aneroid barometer; barograph.
D) an anemograph; an anemometer.

1797. What is the relationship between meteorological visibility (met vis) and RVR in homogeneous
fog?
A) The met. vis is generally less than the RVR.
B) There is no specific relationship between the two.
C) The met. vis generally is the same as the RVR.
D) The met. vis generally is greater than the RVR.

1798. TEMPO TS indicates:


A) TS that will last for at least 30 mins.
B) TS that will last for less than 30 mins.
C) TS that will last for the entire period indicated.
D) TS that will last for a max of 1hr in each instance.

1799. Runway Visual Range (RVR) is:


A) reported in TAF and METAR.
B) reported when meteorological visibility is less than 2000m.
C) usually better than meteorological visibility.
D) measured with ceilometers alongside the runway.

1800. What information is contained in a CONVECTIVE SIGMET?


A) Surface winds greater than 40 knots or thunderstorms equal to or greater
than video integrator processor (VIP) level 4.
B) Severe icing, severe thunderstorms and hail 1/2 inch or greater in
diameter.
C) Tornadoes, embedded thunderstorms, and hail 3/4 inch or greater in
diameter.
D) Severe icing, severe turbulence, or widespread dust storms lowering
visibility to less than 3 miles.
Weather charts:
1801. According to ICAO, which symbol
indicates a tropical revolving storm?
A) Symbol a).
B) Symbol b).
C) Symbol d).
D) Symbol c).

1802. What information is given on a Significant Weather Chart?


A) The significant weather that is observed at the time given on the chart.
B) The significant weather forecast for the time given on the chart.
C) The significant weather forecast for a period 6 hours after the time given
on the chart.
D) The significant weather in a period 3 hours before and 3 hours after the
time given on the chart.

1803. An isohypse of the 500 hPa pressure surface is labelled with the number 552. This means that
for all points on the isohypse the:
A) topography is 552 decimetres above MSL.
B) pressure is 552 hPa.
C) pressure altimeter will overread by 552 FT.
D) topography is 552 meters above MSL.

1805. Contours on a weather chart indicate:


A) distance between pressure levels measured in decametres.
B) thickness between pressure levels.
C) height of ground AMSL.
D) heights of pressure levels AMSL.

1806. According to ICAO, which symbol


indicates danger to an aircraft in flight?
A) Symbol a).
B) Symbol c).
C) Symbol b).
D) Symbol d).
1807. On a significant weather chart you notice a symbol with the letter H and the number 400 inside.
What does this imply?
A) Tropopause high.
B) The height of the significant weather chart.
C) Tropopause low.
D) Tropopause middle.

1808. What positions are connected with contour lines on the weather chart?
A) Positions with the same height in a chart of constant pressure.
B) Positions with the same wind velocity.
C) Positions with the same thickness between two constant pressure levels.
D) Positions with the same air density.

1811. How is the direction and speed of upper winds described in forecasts?
A) The direction is relative to magnetic north and the speed is in miles per hour.
B) The direction is relative to true north and the speed is in knots.
C) The direction is relative to magnetic north and the speed is in knots.
D) The direction is relative to true north and the speed is in miles per hour.

1813. In which meteorological forecast chart is information about CAT regions found?
A) 500 hPa chart.
B) 24 hour surface forecast.
C) Significant Weather Chart.
D) 300 hPa chart.

1814. Which of the following symbols


represents a thunderstorm line?
A) Symbol c
B) Symbol d
C) Symbol a
D) Symbol b

1815. The symbols on the picture, used on Significant


Weather charts represent:
A) quasi stationary front, widespread haze,
Intertropical convergence zone, severe
sand haze.
B) convergence line, tropical cyclone, severe
line squall, widespread sandstorm.
C) upper occluded front, freezing drizzle,
quasi stationary front, blowing sand/dust.
D) Intertropical convergence zone, mountain
waves, moderate turbulence, blowing snow.
1816. In what hPa range is an upper weather chart for FL 340 situated?
A) 400 - 300 hPa
B) 300 - 200 hPa
C) 500 - 400 hPa
D) 600 - 500 hPa

1818. Which constant pressure altitude chart is standard for a 18289 FT pressure level (FL 180)?
A) 700 hPa
B) 300 hPa
C) 200 hPa
D) 500 hPa

1821. On a significant weather chart the thunderstorm symbol signifies:


A) moderate/severe turbulence and moderate/severe icing.
B) severe turbulence and severe icing.
C) moderate turbulence and moderate icing.
D) moderate turbulence and severe icing.

1822. On a significant weather chart you notice a surface weather front with an arrow labelled with the
no. 5 pointing outward perpendicular from the front. This would indicate:
A) front thickness is 5km.
B) front speed is 5kts.
C) front movement is 5nm.
D) front is 5000ft AMSL.

1823. How are well separated CB clouds described on the Significant Weather Chart?
A) EMBD CB.
B) ISOL CB.
C) OCNL CB.
D) FREQ CB.

1824. The 500 mb contour chart is equivalent to Flight Level:


A) FL300
B) FL180
C) FL90
D) FL50

1825. If an Isohypse on a surface pressure chart of 500hPa shows a figure of 522, this indicates:
A) Topography (level) of 522 decametres above MSL.
B) A low surface pressure.
C) Topography (level) of 522m above MSL.
D) Pressure is 522mb.
1826. Which of the following
symbols represents a
tropical revolving storm?
A) Symbol d
B) Symbol c
C) Symbol a
D) Symbol b

1827. Which constant pressure altitude chart is standard for a 30065 FT pressure level (FL 300)?
A) 500 hPa.
B) 200 hPa.
C) 300 hPa.
D) 700 hPa.

1829. When planning a flight at FL 110, which upper wind and temperature chart would be nearest
your flight level?
A) 700 hPa.
B) 500 hPa.
C) 300 hPa.
D) 850 hPa.

1830. The contour chart is a good tool in identifying:


A) upper winds.
B) heights changes.
C) surface winds.
D) density changes.

1831. Which weather chart gives information about icing and the height of the freezing level?
A) 500 hPa chart.
B) Significant weather chart.
C) 700 hPa chart.
D) Surface chart.

1832. If you are planning a flight at FL 290, which of these upper wind and temperature charts would
be nearest your flight level?
A) 500 hPa.
B) 300 hPa.
C) 850 hPa.
D) 700 hPa.
1833. On which of the following aviation weather charts can a pilot most easily find a jetstream?
A) Upper air chart.
B) Surface chart.
C) Significant weather chart.
D) Wind / temperature chart.

1835. Which constant pressure altitude chart is standard for a 9882 FT pressure level (FL 100)?
A) 850 hPa.
B) 500 hPa.
C) 300 hPa.
D) 700 hPa.

1837. The upper wind and temperature chart of 250 hPa corresponds, in a standard atmosphere, to
about:
A) 32.000 FT
B) 30.000 FT
C) 39.000 FT
D) 34.000 FT

1838. According to ICAO, which symbol indicates


danger to an aircraft in flight?
A) Symbols c) and d)
B) Symbol d)
C) Symbols a) and b)
D) Symbol c)

1842. The lines on a contour chart join points of:


A) equal temperature.
B) equal pressure.
C) equal height.
D) equal humidity.

1843. Which constant pressure altitude chart is standard for a 4781 FT pressure level (FL50)?
A) 850 hPa.
B) 500 hPa.
C) 300 hPa.
D) 700 hPa.

1844. According to ICAO, which symbol indicates


severe icing?

A) Symbol d
B) Symbol c
C) Symbol a
D) Symbol b
1849. Which constant pressure altitude chart is standard for a 38662 FT pressure level (FL 390)?
A) 200 hPa.
B) 300 hPa.
C) 700 hPa.
D) 500 hPa.

1850. When planning a flight at FL 60, which upper wind and temperature chart would be nearest your
flight level?
A) 700 hPa
B) 500 hPa
C) 850 hPa
D) 300 hPa

1853. A SIGMET is issued to aircraft in flight giving warning of phenomena on the route ahead. What
distance ahead on the route does the warning cover?
A) 400nm or 2 hours flying time.
B) 300nm or 1 hours flying time.
C) 500nm or 2 hours flying time.
D) 200nm or 1 hours flying time.
Information of flight planning:
1855. What does the term METAR mean?
A) Meteorological terminal area report.
B) Meteorological aerodrome routine report.
C) Aviation routine weather forecast.
D) Aviation routine weather report.

1586. Refer to the following TAF for Zurich. LSZH 261019 20018G30KT 9999 -RA SCT050 BKN080
TEMPO 23012KT 6000 -DZ BKN015 BKN030 BECMG 1518 23020G35KT 4000 RA OVC010= The lowest
visibility forecast at ETA Zurich 1430 UTC is:
A) 6 NM.
B) 4 km.
C) 6 km.
D) 10 km.

1857. With reference to cloud amounts, the abbreviation BKN means:


A) between 1 and 2 octas of clouds present.
B) between 5 and 7 octas of scattered clouds present.
C) between 3 and 4 octas of scattered clouds present.
D) the information is unreliable.

1858. Refer to the following TAF extract: BECMG 1821 2000 BKN004 PROB30 BECMG 2124 0500 FG
VV001 What does the BECMG data indicate for the 18 to 21 hour time frame?
A) The new conditions are achieved between 1800 and 2100 UTC.
B) Many short term changes in the original weather.
C) A quick change to new conditions between 1800 UTC and 1900 UTC.
D) Many long term changes in the original weather.

1859. In which of the following circumstances is a SIGMET issued?


A) A sudden change in the weather conditions contained in the METAR.
B) Fog or a thunderstorm at an aerodrome.
C) Marked mountain waves.
D) Clear ice on the runways of an aerodrome.

1860. Does the following report make sense? LSZH VRB02KT 5000 MIFG 02/02 Q1015 NOSIG
A) The report is nonsense, because it is impossible to observe a
meteorological visibility of 5 km if shallow fog is reported.
B) The report would never be seen, because shallow fog is not reported
when the meteorological visibility is more than 2 km.
C) The report is not possible, because, with a temperature of 2° C and a
dew point of 2° C there must be uniform fog.
D) The report is possible, because shallow fog is defined as a thin layer of
fog below eye level.
1862. Which of the four answers is a correct interpretation of data from the following METAR?
16003KT 0400 R14/P1500 R16/1000N FZFG VV003 M02/M02 Q1026 BECMG 2000 =
A) Meteorological visibility 400m, RVR for runway 16 1000 m, dew point -2°
C, freezing fog.
B) RVR for runway 16 1000 m, meteorological visibility increasing in the
next 2 hours to 2000 m, vertical visibility 300 m, temperature -2° C.
C) RVR for runway 14 1500 m, meteorological visibility 400 m, QNH 1026
hPa, wind 160° at 3 kt.
D) Meteorological visibility 1000 m, RVR 400 m, freezing level at 300 m,
variable winds, temperature 2° C.

1863. In the TAF for Delhi, during the summer, for the time of your landing you note: TEMPO TS.
What is the maximum time this deterioration in weather can last in anyone instance?
A) 60 minutes.
B) 10 minutes.
C) 20 minutes.
D) 120 minutes.

1864. What does the term TREND signify?


A) It is a brief landing forecast added to the actual weather report.
B) It is the actual weather report at an aerodrome and is generally issued at half-
hourly intervals.
C) It is a warning of dangerous meteorological conditions.
D) It is a flight forecast, issued by the meteorological station several times daily.

1865. Which of the following weather reports could be, in accordance with the regulations, abbreviated
to CAVOK?
MSA above ground: LSZB 10000 FT, LSZH 8000 FT, LSGG 12000 FT, LFSB 6000 FT
A) LSZB 30004KT 9999 SCT090 10/09 Q1006 NOSIG =
B) LSZH 26024G52KT 9999 BKN060 17/14 Q1012 RETS TEMPO 5000 TSRA =
C) LFSB 00000KT 9000 SCT080 22/15 Q1022 NOSIG =
D) LSGG 22003KT 9999 SCT120 BKN280 09/08 Q1026 BECMG 5000 =

1866. A METAR from an airfield reporting visibility 8 km. Added to the report is a BECMG 4000. From
this you will expect the visibility to:
A) Be variable between 4 and 8 km.
B) Decrease to 4 km within the next 2 hours.
C) Improve from 4 km to 8 km in the next 2 hours.
D) Decrease to 4 km within 1 hour.

1867. At a weather station, at 0600 UTC, the air temperature and dew point are respectively: T = -
0,5° C, Td = -l,5° C.
In the METAR message transmitted by this station, the temperature group will be:
A) M00/M01
B) 00/M01
C) MO1/M01
D) M01/M02
1870. The TAF validity in terms of time is usually:
A) 9 hours.
B) 12 hours.
C) 6 hours.
D) 3 hours.

1871. Where do you find information on ICING and CAT?


A) SWC and SIGMET.
B) Analysis chart.
C) TAF.
D) METAR.

1872. A pilot is warned of severe icing at certain flight levels by information supplied in:
A) SWC and SIGMET.
B) TAF and METAR.
C) TAF and SIGMET.
D) METAR and SIGMET.

1873. Refer to the following TAF extract:


BECMG 1821 2000 BKN004 PROB30 BECMG 2124 0500 FG W001
What does the abbreviation BKN004 mean?
A) 5 - 7 oktas, ceiling 400 FT.
B) 4 - 8 oktas, ceiling 400 m.
C) 1 - 4 oktas, ceiling 400 FT.
D) 1 - 4 oktas, ceiling 400 m.

1874. In the weather briefing room during the pre-flight phase of a passenger flight from Zurich to
Rome, you examine the following weather reports of pressing importance at the time: EINN SHANNON
2808 sigmet 2 valid 0800/1100 loc sev turb fcst einn fir blw fl 050 south of 53n wkn = LIMM MILANO
2809 sigmet 2 valid 0900/1500 mod sev cat btn fl 250 and fl 430 fcst limm fir stnr nc = EGLL
LONDON 2808 sigmet nr01 valid 0800/1200 for london fir isol cb embd in lyr cloud fcst tops fl 300 btn
52n and 54n east of 002e sev ice sev turb ts also fcst mov e wkn = Which decision is correct?
A) Because of the expected turbulence you select a flight level below FL 250.
B) You show no further interest in these reports, since they do not concern
the route to be flown.
C) You cancel the flight since the expected dangerous weather conditions
along the route would demand too much of the passengers.
D) Owing to these reports and taking into account the presence of heavy
thunderstorms at planned FL 310 you select a higher flight level (FL 370).

1875. The information about visibility to be used planning a VFR flight is:
A) Observed horizontal visibility on ground.
B) Observed vertical visibility.
C) RVR.
D) Forecasted horizontal visibility on ground.
1877. What is a trend forecast?
A) An aerodrome forecast valid for 9 hours.
B) A route forecast valid for 24 hours.
C) A landing forecast appended to METAR/SPECI, valid for 2 hours.
D) A routine report.

1878. VRB, BR and = mean respectively:


FT1756 KEWR 291715Z 291818 04010KT 1 1/2SM -RA BR OVC003
TEMPO1824 1/2SM -RADZ FG VV001
BECMG2224 VRB03KT
FM0800 29010KT 2SM BR OVC008
TEMPO 0814 3SM BR OVC012
FM1400 30013G23KT P6SM SCT012 OVC025
TEMPO 1416 4SM -SHSN BR BKN012
FM1600 30017G32KT P6SM BKN 035 PROB30 1618 4SM -SHSN BR=
A) variable wind direction; breezes; end of TAF.
B) variable wind speed; breezes; no TREND on this TAF.
C) variable wind direction; blowing snow; end of TAF.
D) variable wind direction; mist; end of TAF.

1880. Which of the following is a landing forecast?


A) METAR.
B) TAF.
C) SPECI.
D) METAR with TREND.
1881. What units are used to report vertical wind shear?
A) m/100 FT.
B) kt.
C) m/sec.
D) kt/100 FT.

1883. Refer to the TAF for Amsterdam airport. FCNL31 281500 EHAM 281601 14010KT 6000 -RA
SCT025 BECMG 1618 12015G25KT SCT008 BKN013 TEMPO 1823 3000 RA BKN005 OVC010 BECMG
2301 25020KT 8000 NSW BKN020 = Flight from Bordeaux to Amsterdam, ETA 2100 UTC. What lowest
cloud base is forecast for arrival at Amsterdam?
A) 250 FT
B) 500 m
C) 800 FT
D) 500 FT

1886. What does the abbreviation nosig mean?


A) No weather related problems.
B) Not signed by the meteorologist.
C) No significant changes.
D) No report received.
1887. What is the meaning of the abbreviation SCT?
A) 1 - 2 oktas.
B) 3 - 4 oktas.
C) 1 - 4 oktas.
D) 5 - 7 oktas.

1889. When will the surface wind in a METAR record a gust factor?
A) With gusts of at least 25 knots.
B) With gusts of at least 35 knots.
C) When gusts are at least 15 knots above the mean wind speed.
D) When gusts are at least 10 knots above the mean wind speed.

1890. SIGMETs are issued as a warning of weather conditions hazardous to which aircraft?
A) All aircraft.
B) All IFR traffic.
C) Small aircraft only.
D) Large aircraft only.

1891. Refer to the TAF for Zurich Airport TAF LSZH 250716 00000KT 0100 FG VV001 BECMG 0810
0800 VV002 BECMG 1012 23005KT 2500 BKN005 TEMPO 1316 6000 SCT007 = Which of these
statements best describes the weather that can be expected at 1200 UTC?
A) Meteorological visibility 800 metres, vertical visibility 200 feet, calm.
B) Meteorological visibility 800 metres, wind from 230° , cloud base 500 feet.
C) Meteorological visibility 2,5 kilometres, cloud base 500 feet, windspeed 5 knots.
D) Meteorological visibility 6 kilometres, cloud base 500 feet, windspeed 5 knots.

1892. A flight is to depart from an airport with runways 09 and 27. Surface wind is 270/05; an
inversion is reported at 300 feet with turbulence and wind shear. The wind just above the inversion is
090/30. What is the safest departure procedure?
A) Depart on runway 09 with a tailwind.
B) Depart runway 27 with as steep an ascent as possible.
C) Depart runway 27 with maximum throttle, during the passage through the inversion.
D) Take-off is not possible under these conditions.

1893. In METAR messages, the pressure group represents the:


A) QFE rounded to the nearest hPa.
B) QNH rounded down to the nearest hPa.
C) QNH rounded up to the nearest hPa.
D) QFE rounded down to the nearest hPa.

1894. Runway visual range can be reported in:


A) a METAR.
B) a TAF.
C) both a TAF and a METAR.
D) a SIGMET.
1895. What OAT would you expect at FL 200 over Geneva?
A) -20° C
B) -24° C
C) -16° C
D) -28° C

1896. What is the meaning of the abbreviation BKN?


A) 6 - 8 octas.
B) 3 - 4 octas.
C) 8 octas.
D) 5 - 7 octas.

1897. A pilot experiences severe turbulence and icing. A competent met. man would issue:
A) TEMPO.
B) SIGMET.
C) METAR.
D) SPECI.

1898. The following are included in supersonic level SIGMETS:


A) mod/severe turbulence, thunderstorms, mod/severe. icing, hail.
B) turbulence, thunderstorms, icing, volcanic ash.
C) mod/severe turbulence, CB cloud, hail, volcanic ash.
D) severe turbulence, severe icing, thunderstorms, icing, severe mountain
waves.

1899. If weather conditions are reported as CAVOK, which of the following could not be present:
A) Overcast cloud at 5100ft.
B) 1 oktas of CB at 9000ft.
C) Winds 25 KT, gusting to 40 KT.
D) All above mentioned cases can exist for CAVOK reported conditi

1900. What is indicated when a current CONVECTIVE SIGMET forecasts thunderstorms?


A) Severe icing.
B) Thunderstorms obscured by massive cloud layers.
C) Moderate or severe turbulence.
D) Moderate thunderstorms covering 30 percent of the area.

1901. How may the correct wind speed be found, for a level, which is between two upper air chart
levels? (e.g. wind at FL 250, when the 500 hPa and the 300 hPa chart are available).
A) By simple interpolation of wind information available from the two charts.
B) By interpolation of the wind information available from the two charts,
while also considering the maximum wind information found on the
Significant Weather Chart.
C) By reading wind direction and speed from the next higher chart.
D) By reading wind direction and speed from the 300 hPa chart.
1902. SIGMET information is issued as a warning for significant weather to:
A) VFR operations only.
B) heavy aircraft only.
C) light aircraft only.
D) all aircraft.

1903. In which weather report would you expect to find information about icing conditions on the
runway?
A) GAFOR
B) TAF
C) SIGMET
D) METAR

1904. In which of the following METAR reports, is the probability of fog formation in the coming night
the highest?
A) 1850Z 25010KT 4000 RA BKN012 OVC030 12/10 Q1006 TEMPO 1500 =
B) 1850Z 15003KT 6000 SCT120 05/04 Q1032 BECMG 1600 =
C) 1850Z 21003KT 8000 SCT250 12/m08 Q1028 NOSIG =
D) 1850Z 06018G30KT 5000 OVC010 04/01 Q1024 NOSIG =

1905. What would the code 01650428 attached to a METAR tell you about the condition of the
runway?
A) Its broken.
B) Its snowing.
C) Braking co-efficient of 0.28
D) Its raining.

1906. The surface wind is given by Air Traffic Control as 300° /40 kts and variation at the airfield is
20° W. Runway 24 is in use; what is the headwind component and the direction of the crosswind
component?
A) 31 kts; left to right.
B) 20 kts; left to right.
C) 35 kts; right to left.
D) 20 kts; right to left.

1907. Which of the following statements is an interpretation of the METAR? 00000KT 0200 R14/0800U
R16/P1500U FZFG VV001 m03/m03 Q1022 BECMG 0800 =
A) Meteorological visibility 200 feet, RVR for runway 16 more than 1500
metres, vertical visibility 100 feet, fog with hoar frost.
B) Meteorological visibility 200 metres, RVR for runway 16 1500 metres,
temperature -3° C, vertical visibility 100 metres.
C) RVR for runway 14 800 metres, vertical visibility 100 feet, calm,
meteorological visibility improving to 800 metres in the next 2 hours.
D) Meteorological visibility for runway 14 800 metres, fog with hoar frost,
RVR for runway 16 more than 1500 metres.
1908. LLWAS is:
A) a low level warning system for the detection of thunderstorms.
B) a North American system for the detection of Marked Temperature Inversion.
C) a North American system for giving warning of low level windshear.
D) a North American radar network for the detection of tornadoes.

1910. If CAVOK is reported then:


A) any CBs have a base above 5000 FT.
B) no clouds are present.
C) low level windshear has not been reported.
D) no low drifting snow is present.

1913. Which of the following phenomena should be described as precipitation at the time they are
observed?
A) FZFG
B) HZ
C) SN
D) BCFG

1914. How is the cloud base reported in a METAR?


A) In steps of 10 ft up to 10000 ft and in steps of 1000 ft above 10000 ft
B) In steps of 10 m up to 10000 ft and in steps of 1000 ft above 10000 ft
C) In steps of 100 m up to 10000 ft and in steps of 1000 ft above 10000 ft.
D) In steps of 100 ft up to 10000 ft and in steps of 1000 ft above 10000 ft.

1917.SIGMETs are issued as a warning of weather conditions hazardous:


A) Only to light aircraft operations.
B) To all aircraft.
C) Particularly to light aircraft.
D) Particularly to heavy aircraft.

1919.Refer to the TAF below: What are the weather conditions you have to expect at 1200UTC on
Zurich airport?
TAF LSZH 0716 00000KT 0100FG VV001 BECMG 0810 0800 VV002 BECMG 1012 23005KT 2500
BKN005 TEMPO 1316 6000 SCT007 =
A) met. vis. 6 km, ceiling 500 ft, wind from 230 degrees.
B) met. vis. 800 m, wind direction 230 degrees, ceiling 500 ft.
C) met. vis. 800 metres, vertical vis. 200ft, calm.
D) met. vis. 2,5 km, ceiling 500 ft, wind speed 5 KT.

1921. The RVR, as reported in a METAR, is always the:


A) highest value of the A-, B- and C-position.
B) average value of the A-, B- and C-position.
C) lowest value of the A-, B- and C-position.
D) value representative of the touchdown zone.
1924. The wind direction in a METAR is measured relative to:
A) true north.
B) grid north.
C) the 0-meridian.
D) magnetic north.

1925. Refer to TAF below. EGBB 261812 28015G25KT 9999 SCT025 TEMPO 1822 29018G35KT 5000
SHRASN BKN010CB PROB30 TEMPO 1821 1500 TSGR BKN008CB BECMG 2124 26010 KT From the
TAF above you can assume that visibility at 2055Z in Birmingham (EGBB) will be:
A) a maximum 5 km.
B) more than 10 km.
C) a minimum of 1,5 km and a maximum of 5 km.
D) not less than 1,5 km but could be in excess of 10 km.

1926. Refer to the TAF for Bordeaux airport.


FCFR31 281400 LFBD 1524 26015KT 9999 SHRA BKN020 TEMPO 1620 26020G30KT 8000 +SHRA
BKN015CB PRQB30 TSRA =
Flight Lisbon to Bordeaux, ETA 1800 UTC. What type of precipitation is forecast on the approach to
Bordeaux?
A) Continuous moderate rain.
B) Heavy rain showers.
C) Light drizzle and fog.
D) Moderate snow showers.

1927. In which of the following 1850 UTC METAR reports, is the probability of fog formation, in the
coming night, the highest?
A) VRB02KT 2500 SCT120 14/M08 Q1035 NOSIG =
B) 00000KT 9999 SCT300 21/01 Q1032 NOSIG =
C) 22004KT 6000 -RA SCT012 OVC030 17/14 Q1009 NOSIG =
D) VRB01KT 8000 SCT250 11/10 Q1028 BECMG 3000 =

1928. A SPECI is:


A) an aviation routine weather report.
B) a warning for special weather phenomena.
C) an aviation selected special weather report.
D) a forecast for special weather phenomena.

1929. Cloud bases in TAFs and METARs are given in:


A) Heights above mean sea level (TAFs) and heights above airfield elevation
(METARs)
B) Heights above airfield elevation.
C) Heights above the 1013 hPa pressure level.
D) Heights above mean sea level.
1930. For your flight from Zurich to Rome you consult, amongst other information, the following
SIGMET: SIGMET VALID121420/121820 MOD TO SEV CAT OBS AND FCST S OF ALPS BTN FL 260 AND
FL 380 / STNR / INTSF =
A) You select a flight level between FL 260 and FL 380 due to turbulence.
B) You delay your departure until 18.20 LT to avoid the icing conditions
between FL 260 and FL 380.
C) You disregard this SIGMET, because it does not affect your flight.
D) You select a flight level below FL 260 due to turbulence.

1931. Which of these four METAR reports suggests that a thunderstorm is likely in the next few hours?
A) 1350Z 04012KT 3000 OVC012 04/03 Q1022 BECMG 5000 =
B) 1350Z 16004KT 8000 SCT110 OVC220 02/m02 Q1008 NOSIG =
C) 1350Z 21005KT 9999 SCT040CB SCT100 26/18 Q1016 TEMPO 24018G30 TS =
D) 1350Z 34003KT 0800 SN VV002 m02/m04 Q1014 NOSIG =

1933. What conditions have to be met amongst others, that the weather is reported to be CAVOK?
A) No clouds are present below 8000 ft AGL.
B) No clouds are present below 5000 ft AMSL.
C) No low drifting snow is present.
D) Wind of less than 25KT.

1934. A TREND is a ... and is added to...


A) change in weather (BECMG); a TAF.
B) valid for 2 hours; a METAR or SPECI.
C) temporary change; a TAF.
D) 2 hour forecast; a SIGMET.

1935. The validity of a TAF is:


A) 2 hours
B) stated in the TAF.
C) 9 hours from the time of issue.
D) between 6 and 9 hours.

1936. When the term INTER occurs in a trend forecast it means:


A) The following intermediate conditions apply.
B) A change is expected to occur in a short period of time.
C) Changes are expected to occur frequently for short periods of time.
D) Changes are expected to last for a period of less than 1 hour in each
instant.

1937. Given the following METAR: EDDM 250850Z 33005KT 2000 R26R/P1500N R26L/1500N BR
SCT002 OVC003 05/05 Q1025 NOSIG
A) RVR on runway 26R is increasing.
B) Visibility is reduced by water droplets.
C) Runway 26R and runway 26L have the same RVR.
D) There is a distinct change in RVR observed.
1938. Where are icing conditions on a runway specified?
A) SIGMET
B) TAF
C) GAFFO
D) METAR

1939. How long from the time of observation is a TREND in a METAR valid?
A) 2 hours.
B) 9 hours.
C) 1 hour.
D) 30 minutes.

1941. Which of the statements is true concerning squall lines?


A) Severe squall lines only occur in the tropics.
B) Severe squall lines always move from northwest to southeast.
C) For severe squall lines a SIGMET is issued.
D) For severe squall lines a TAF is issued.

1943. TAF KPIT 14100Z 1212 10013KT 0800 DZ W004 TEMPO 1214 2400 – RA OVC008 BECMG 1416
03012KT 3200 – RA OVC010 TEMPO 1620 1200 +RA OVC005 PROB40 TEMPO 1618 0800 TS VV002
BECMG 1820 4800 SCT008 OVC015 TEMPO 2024 3200 – RA OVC008 PROB40 TEMPO 2024 1200 TS
OVC005CM BECMG 0003 SCT025 TEMPO 0307 1600 BR BKN006 BECMG 0709 0800 FG VV003= The
forecast surface visibility at PITTSBURGH (KPIT) at 2100 UTC is:
A) Changing frequently from 4.800 m to 3.200 m, 1.200 m and back to 4.800 m.
B) Changing steadily from 4.800 m to 3.200 m.
C) 3.200 m
D) 4.800 m

1944. What does the term SIGMET signify?


A) A SIGMET is a flight forecast, issued by the meteorological station several times daily.
B) A SIGMET is a warning of dangerous meteorological conditions.
C) A SIGMET is a brief landing forecast added to the actual weather report.
D) A SIGMET is an actual weather report at an aerodrome and is generally issued
at half-hourly intervals.

1945. TAF LSZH 1322 22018G35KT 9999 SCT012 BKN030 BECMG 1315 25025G45KT BECMG 1820
4000 RA BKN025 BECMG 2022 25015KT T1815Z T1618Z = Which statement is true at 15 UTC?
A) Met. vis. 10 km or more, ceiling 3000 ft, wind from 250 degrees, temperature 18° C.
B) Met. vis. 4000 metres, gusts of 25 KT, temperature 18° C.
C) Strong rain showers, met. vis. 4000 metres, temperature 15° C, gusts of 35 KT.
D) Met. vis. 10 km or more, ceiling 1200 ft, gusts of 45 KT.

1946. Refer to the following TAF extract: BECMG 1821 2000 BKN004 PROB30 BECMG 2124 0500 FG
VV001 What does the abbreviation VV001 mean?
A) Vertical visibility 100 m.
B) RVR less than 100 m.
C) RVR greater than 100 m.
D) Vertical visibility 100 FT.
1947. Appended to a METAR you get the following runway report: 01650428 What must you consider
when making performance calculations?
A) Aquaplaning conditions.
B) The friction coefficient is 0.28.
C) The braking action will be medium to good.
D) The runway will be wet.

1948. The function of a TAF is to be a/an:


A) Wind forecast.
B) Local forecast.
C) Forecast for an airfield.
D) Area forecast.

1949. Which of these four METAR reports suggests that rain is most likely in the next few hours?
A) 34004KT 9999 SCT040 SCT100 m05/m08 Q1014 NOSIG =
B) 16002KT 0100 FG SCT300 06/06 Q1022 BECMG 1000
C) 23015KT 8000 BKN030 OVC070 17/14 Q1009 BECMG 4000 =
D) 05016G33KT 8000 OVC015 08/06 Q1028 NOSIG =

1950. You receive the following METAR: LSGG 0750Z 00000KT 0300 R05/0700N FG VV001 M02/M02
Q1014 NOSIG = What will be the RVR at 0900 UTC?
A) 700 m.
B) 300 m.
C) 900 m.
D) The RVR is unknown, because the NOSIG does not refer to RVR.

1952. Select from the map the average temperature for the route Geneva -Stockholm at FL 260.
A) -63° C
B) -47° C
C) -51° C
D) -55° C

1953. EHAM 120600Z 02025KT 3000 RA BKN005 OVC015 11/10 Q1012= Which of the following is the
correct decode for the Amsterdam METAR?
A) Observation at 0600 UTC, wind 020° at 25 knots, visibility 3000m, rain, broken
ST with base 500ft AGL and 8 oktas NS with base 1500ft MSL, temperature
+11° C, dew point +10° C, QNH 1012 hPa.
B) Period valid between 0600 UTC and 0000 UTC, wind 020° at 25 knots, visibility
3000 metres in rain, clouds: broken ST with base 500ft AGL and 8 oktas NS with
base 1500ft AGL, temperature +11° C, dew point +10° C, QNH 1012 hPa.
C) Observation at 0600Z, wind 020° at 25 knots, visibility 3000 metres, rain,
clouds: 3 oktas ST with base 500ft AGL and 8 oktas NS with base 1500ft AGL,
temperature +11° C, dew point +10° C, QNH 1012 hPa.
D) Observation at 0600Z, wind 020° at 25 knots, visibility 3000 metres in rain,
clouds: broken ST with base 500ft AGL and 8 oktas NS with base 1500ft AGL,
temperature +11° C, dew point +10° C, QNH 1012 hPa.
1954. ATIS information contains:
A) operational information and if necessary meteorological information.
B) meteorological and operational information.
C) only meteorological information.
D) only operational information.

1955. Refer to the TAF for Amsterdam airport. FCNL31 281500 EHAM 281601 14010KT 6000 -RA
SCT025 BECMG 1618 12015G25KT SCT008 BKN013 TEMPO 1823 3000 RA BKN005 OVC010 BECMG
2301 25020KT 8000 NSW BKN020 = Flight from Bordeaux to Amsterdam, ETA 2100 UTC. At ETA
Amsterdam what surface wind is forecast?
A) 140° / 10 kt
B) 250° / 20 kt
C) 300° / 15 kt maximum wind 25 kt
D) 120° / 15 kt gusts 25 kt

1958.Refer to the TAF for Amsterdam airport. FCNL31 281500 EHAM 281601 14010KT 6000 -RA
SCT025 BECMG 1618 12015G25KT SCT008 BKN013 TEMPO 1823 3000 RA BKN005 OVC010 BECMG
2301 25020KT 8000 NSW BKN020 = Flight from Bordeaux to Amsterdam, ETA 2100 UTC. What is the
minimum visibility forecast for ETA Amsterdam?
A) 6 km.
B) 3 km.
C) 5 NM.
D) 5 km.

1959. TAF KPIT 14100Z 1212 10013KT 0800 DZ W004 TEMPO 1214 2400 – RA OVC008 BECMG 1416
03012KT 3200 – RA OVC010 TEMPO 1620 1200 +RA OVC005 PROB40 TEMPO 1618 0800 TS VV002
BECMG 1820 4800 SCT008 OVC015 TEMPO 2024 3200 – RA OVC008 PROB40 TEMPO 2024 1200 TS
OVC005CM BECMG 0003 SCT025 TEMPO 0307 1600 BR BKN006 BECMG 0709 0800 FG VV003= The
forecast low cloud at PITTSBURGH (KPIT) at 2300 UTC is likely to be:
A) 6/8, base 500ft CB.
B) 7/8, base 800ft.
C) 3/8, base 800ft.
D) B and C could be present

1960. In which of the following METARs is the probability the biggest for the formation of fog the
following night?
A) 1850Z 15003KT 6000 SCT120 05/04 Q1032 BECMG 1600 =
B) 1850Z 25010KT 4000 RA BKN012 OBC030 12/10 Q1006 TEMPO 1500 =
C) 1850Z 21003KT 8000 SCT250 12/M08 Q1028 NOSIG =
D) 1850Z 06018G30KT 5000 OVC010 04/01 Q1024 NOSIG =
1961. At DELHI (VIDP) when might thunderstorms be expected?

A) At any time between 0200 and 0600.


B) At any time between 1300 and 1500.
C) At any time between 1200 and 0400.
D) At any time between 1200 and 0600.

1962. Which of the following weather reports could be, in accordance with the regulations, abbreviated
to CAVOK?
MSA above ground: LSZB 10000 FT, LSZH 8000 FT, LSGG 12000 FT, LFSB 6000 FT
A) LSZH VRB02KT 9000 BKN080 21/14 Q1022 NOSIG =
B) LFSB 24008KT 9999 SCT050 18/11 Q1017 RERA NOSIG =
C) LSGG 22006KT 9999 BKN090 17/15 Q1008 RERA NOSIG =
D) LSZB 28012KT 9999 OVC100 16/12 Q1012 BECMG 5000

1965. A SIGMET is:


A) A message about observed or forecasted weather phenomena of special
importance to aviation.
B) A special METAR.
C) The same as a TAF forecast.
D) A significant weather chart.

1966. When would a SIGMET be issued for subsonic flights?


A) Severe Mountain Waves.
B) Moderate turbulence.
C) Solar Flare activity.
D) Thunderstorms and fog.

1967. In a METAR a gust is reported when:


A) it is 15 kts greater than the mean wind speed.
B) it is 5 kts greater than the mean wind speed.
C) it is 10 kts greater than the mean wind speed.
D) it is 20 kts greater than the mean wind speed.
1968. TAF KPIT 14100Z 1212 10013KT 0800 DZ W004 TEMPO 1214 2400 – RA OVC008 BECMG 1416
03012KT 3200 – RA OVC010 TEMPO 1620 1200 +RA OVC005 PROB40 TEMPO 1618 0800 TS VV002
BECMG 1820 4800 SCT008 OVC015 TEMPO 2024 3200 – RA OVC008 PROB40 TEMPO 2024 1200 TS
OVC005CM BECMG 0003 SCT025 TEMPO 0307 1600 BR BKN006 BECMG 0709 0800 FG VV003= The
lowest vertical visibility at PITTSBURGH (KPIT) will probably be:
A) between 1600 and 1800 UTC.
B) between 1200 and 1400 UTC.
C) between 2000 and 2400 UTC.
D) at 0800 UTC.

1969. Which of the following phenomena can produce a risk of aquaplaning?


A) BCFG
B) +RA
C) SA
D) FG

1970. Which of the following weather reports is a warning of conditions that could be potentially
hazardous to aircraft in flight?
A) TAF.
B) ATIS.
C) SIGMET.
D) SPECI.

1971. In TAF and METAR decodes, DU, DZ and DS mean respectively:


A) dust, dust devils, dust storms.
B) dust devils, drizzle, dust and sand.
C) dust, drizzle, dust storms.
D) dust, drizzle, dust and sand.

1972. Which of the following statements is an interpretation of the SIGMET? SIGMET VALID
121420/121820 embd ts obs and fcst in w part of athinai fir / mov e / intst nc =
A) Thunderstorms must be expected in the western part of the Athens FIR.
The thunderstorm zone is moving east. Intensity is constant.
B) The thunderstorms in the Athens FIR are increasing in intensity, but are
stationary above the western part of the Athens FIR.
C) Athens Airport is closed due to thunderstorms. The thunderstorm zone
should be east of Athens by 1820 UTC.
D) Thunderstorms have formed in the eastern part of the Athens FIR and are
slowly moving west.

1973. A SIGMET message concerns:


A) severe weather that is encountered by the pilot of an aircraft.
B) weather conditions likely to affect the safety of aircraft operations issued by a
local Met Office.
C) weather phenomena likely to affect the safety of aircraft operations issued by a
Meteorological Watch office.
D) a routing report of sig weather issued by a local Met Office.
1974. If a special route forecast is required for a sector length of 450nm the notice required by the
forecast office is:
A) 4 hours before departure.
B) 2 hours before collection.
C) 2 hours before departure.
D) 4 hours before collection.

1975. The SIGMET for OEJD indicates:

A) rising sand reducing the visibility in the NE of JEDDAH FIR to 3-5 km


locally 1.000 m up to 4.000 ft afl, visibility expected to improve.
B) sand storms reducing visibility in JEDDAH FIR to 1.000m.
C) rising sand reducing visibility in the NE of JEDDAH FIR to 3-5 km locally
1.000 m up to 4.000 ft amsl, visibility expected to deteriorate.
D) heavy sand storm reducing visibility in the NE of JEDDAH FIR to 3-5 km
locally 1.000 m up to FL040, visibility expected to deteriorate.

1976. Which of the following METARs, written at 1850UTC, will most likely give fog formation over the
coming night?
A) VRB002 9999 SCT150 17/M08 Q1012 NOSIG=
B) VRB001 8000 SCT280 11/10 Q1028 BECMG 3000
C) 240/04 6000 -RA SCT012 OVC 3000 17/14 Q1002 NOSIG=
D) VRB002 8000 FEW100 12/09 Q1025 BECMG 0800

1978. Within a short interval, several flight crews report that they have experienced strong clear air
turbulence in a certain airspace. What is the consequence of these reports?
A) The competent aviation weather office will issue a SIGMET.
B) The competent aviation weather office will issue a SPECI.
C) The airspace in question, will be temporarily closed.
D) The competent aviation weather office will issue a storm warning.
1979. Which of the following statements is an interpretation of the METAR? 25020G38KT 1200 +TSGR
BKN006 BKN015CB 23/18 Q1016 BECMG NSW =
A) Wind 250° , thunderstorm with moderate hail, QNH 1016 hPa.
B) Gusts of 38 knots, thunderstorm with heavy hail, dew point 18° C.
C) Broken, cloud base 600 feet and 1500 feet, temperature 18° C.
D) Mean wind speed 20-38 knots, meteorological visibility 1200 metres,
temperature 23° C.

1980. EGCC 150820Z VRB03KT 3500N FU SKC 08/07 Q1024 TEMPO 0800 BCFG
What type of weather message is this?
A) Trend-type METAR
B) METAR
C) 9-hour TAF
D) 16-hour TAF

1981. The abbreviation FEW means:


A) 1-3 oktas of cloud cover.
B) 5-7 oktas cloud cover.
C) 1-2 oktas cloud cover.
D) 2-4 oktas cloud cover.

1982. Which of the following phenomena should be described as precipitation at the time they are
observed?
A) VA
B) BR
C) MIFG
D) +SHSN

1985. The heights of cloud bases in TAFs are reported as being:


A) AAL
B) AGL
C) AMSL
D) Pressure altitude.

1986. The airborne system for reporting weather data automatically to the WAFCs is called:
A) AWACS.
B) CARS.
C) ASDAR.
D) AIREP.

1987. A SIGMET...
A) reports the actual weather of an airport every 30 minutes.
B) is a warning of meteorological hazards in the FIR for all aircraft.
C) is a route forecast, issued several times during the day by the Met Offices.
D) is added as a short-term forecast to the actual weather report of an airport.
1988. Marseille Information gives you the following meteorological information for Ajaccio
and Calvi for 16:00 UTC:
Ajaccio: wind 360° /2 kt, visibility 2000 m, rain, BKN stratocumulus at 1000 FT, OVC
altostratus at 8000 FT, QNH 1023 hPa. Calvi: wind 040° /2 kt, visibility 3000 m, mist, FEW
stratus at 500 FT, SCT stratocumulus at 2000 FT, OVC altostratus at 9000 FT, QNH 1023
hPa. The ceilings (more than 4 oktas) are therefore:
A) 1000 FT at Ajaccio and 9000 FT at Calvi.
B) 1000 FT at Ajaccio and 2000 FT at Calvi.
C) 8000 FT at Ajaccio and 9000 FT at Calvi.
D) 1000 FT at Ajaccio and 500 FT at Calvi.

1989. On the European continent METARs of main airports are compiled and distributed with
intervals of:
A) 2 hours.
B) 3 hours.
C) 0.5 hour.
D) 1 hour.

1990. In the TAF for OEDR the group NSC indicates:

A) no CB, no cloud below 5.000 ft or MSA whichever is lesser and visibility more than 10 km
B) no CB or clouds below 5.000 ft or MSA whichever is greater, and CAVOK or SKC are not
appropriate.
C) no cloud except CB and visibility 10 km or more.
D) no cloud below 5.000 ft or MSA but SKC is not appropriate.

1991. Which of the following phenomena should be described as precipitation at the time
they are observed?
A) SA
B) TS
C) SQ
D) DZ

1992. Which of the following weather reports could be, in accordance with the regulations,
abbreviated to CAVOK?
A) 04012G26KT 9999 BKN030 11/07 Q1024 NOSIG =
B) 15003KT 9999 BKN100 17/11 Q1024 NOSIG =
C) 29010KT 9999 SCT045TCU 16/12 Q1015 RESHRA NOSIG =
D) 24009KT 6000 RA SCT010 OVC030 12/11 Q1007 TEMPO 4000 =
1993. What does the expression Broken (BKN) mean?
A) 3-5 Eights of the sky is cloud covered.
B) 5-7 Eights of the sky is cloud covered.
C) Nil significant cloud cover.
D) 3-4 Eights of the sky is cloud covered.

1995. Refer to the following TAF extract; BECMG 1821 2000 BKN004 PROB30 BECMG 2124 0500 FG VV001 What
does the abbreviation PROB30 mean?
A) Change expected in less than 30 minutes.
B) Probability of 30%.
C) The cloud ceiling should lift to 3000 FT.
D) Conditions will last for at least 30 minutes.

1996. Refer to the following TAF for Zurich. LSZH 061019 20018G30KT 9999 -RA SCT050 BKN080 TEMPO 23012KT
6000 -DZ BKN015 BKN030 BECMG 1518 23020G35KT 4000 RA OVC010= The lowest cloud base forecast at ETA
Zurich (1200 UTC) is:
A) 1500 m.
B) 1500 FT.
C) 1000 FT.
D) 5000 FT.

1998. The validity period for this TAF is ... and 1 1/2SM means ...:
FT1756 KEWR 291715Z 291818 04010KT 1 1/2SM -RA BR OVC003 TEMPO1824 1/2SM -RADZ FG VV001
BECMG2224 VRB03KT FM0800 29010KT 2SM BR OVC008 TEMPO 0814 3SM BR OVC012FM1400 30013G23KT
P6SM SCT012 OVC025 TEMPO 1416 4SM -SHSN BR BKN012
FM1600 30017G32KT P6SM BKN 035 PROB30 1618 4SM -SHSN BR=
A) 291700Z to 301500Z; best visibility 11SM, worst 1/2SM.
B) 291800Z to 301800Z; visibility one and a half SM.
C) 291800EST to 301800EST; visibility one and a half SM.
D) 291756Z for 12hr; visibility half a statute mile.

2000. In the following METAR: METAR YUDO 1630Z 24015KMH 0600 R12/1000U FG DZ SCT010 17/16 Q1018:
A) airfield visibility is 600m, runway visual range is a maximum of 1000m on touchdown on R/W 12, weather is fog
and drizzle.
B) W/V is 240(T) at 15 kilometres per hour airfield visibility is 600m, runway visual range at touchdown on R/W 12
is 1000m showing a tendency to improve in the last 10 minutes, QNH is 1018 hPa.
C) W/V is 240 deg (M) at 15 kt, runway visual range at touchdown on R/W 12 is 1000m showing a tendency to
improve in the last 10 minutes, dewpoint is +16 deg C.
D) W/V is 240 deg (T) at 15 kilometres per hour, cloud is 4/8 at 100 ft, QNH is 1018 hPa.

2001.For a landing at 301500Z the worst and most probable visibilities you might see are:
FT1756 KEWR 291715Z 291818 04010KT 1 1/2SM -RA BR OVC003
TEMPO1824 1/2SM -RADZ FG VV001
BECMG2224 VRB03KT
FM0800 29010KT 2SM BR OVC008
TEMPO 0814 3SM BR OVC012
FM1400 30013G23KT P6SM SCT012 OVC025
TEMPO 1416 4SM -SHSN BR BKN012
FM1600 30017G32KT P6SM BKN 035 PROB30 1618 4SM -SHSN BR=
A) 6 statute miles or more; 4 statute miles
B) 4 statute miles; 4 statute miles
C) 4 statute miles; 5 statute miles
D) 4 statute miles; 6 statute miles or more

You might also like